Genetics, PEDS GENETICS AND METABOLISM

¡Supera tus tareas y exámenes ahora con Quizwiz!

karyotype for turner syndrome

(45,X; 45,X/46,XX)

What is the inheritance pattern for Type 1 glycogen storage disease (von Gierke disease)?

Autosomal recessive

Turner syndrome findings

(short stature, lymphedema, wide-spaced nipples, heart murmur, and high blood pressure) is most consistent with Turner syndrome (TS).

What lab would you order if worried about CAH? congenital adrenal hyperplasia (virilization, salt-wasting crisis).

17-OH progesterone level

Associations w/ chromosomal anomalies

47,XX,+21; trisomy 21 - transient myeloproliferative disorder (TMD) Hypoplastic and fused cervical vertebrae is a component of Klippel-Feil syndrome. Polydactyly is associated with trisomy 13 (a.k.a. Patau syndrome). absent radius and thumb is often observed in Holt-Oram syndrome. Holt-Oram syndrome is also referred to as heart-hand syndrome, as these are the most commonly affected. Unilateral or bilateral asymmetric upper limb (radius, thumb, carpal bones) abnormalities associated with an atrial septal or ventricular septal defect are consistent with Holt-Oram syndrome. congenital focal absence of the skin in several areas of the scalp, consistent with aplasia cutis congenita (ACC). trisomy 13. Poland sequence, characterized by unilateral absence or hypoplasia of the medial (sternal) portion of the pectoralis major muscle.Associated conditions may include dextrocardia, fused cervical vertebrae (Klippel-Feil syndrome), or congenital facial paralysis and limb reduction defects (Möbius syndrome). NF1 is caused by a mutation of a gene on the long arm of chromosome 17

When would you order a biotinidase level on a patient?

A biotinidase level would be indicated for a child presenting with developmental regression, eczema, hypotonia, ataxia, vision problems, hearing loss, alopecia, and seizures in association with a biotinidase deficiency.

What labs would you order if worried about inborn errors of metabolism?

A serum analysis for: •Acylcarnitine panel •Amino acids •Ammonia •Blood glucose •Carnitine profile •Electrolytes •Lactate •Pyruvate Urinalysis for: •organic acids •ketones

Maternally acquired chromosome 15 anomaly

Angelman syndrome - MA

A 6-month-old boy, new to your practice, is noted to have fusion of the index, middle, ring, and 5th fingers and a common nail bed. A plain radiograph confirms that the involved fingers share fused bone and soft tissue components. The thumb is relatively well formed and has full range of motion. A similar deformity is present in both feet. Which of the following is most likely to be identified upon additional evaluation of this patient? A Bicoronal synostosis and maxillary hypoplasia B Aplasia cutis congenita and limb reduction defects C Dysplastic sacrum and multiple hemivertebrae D Dysplastic patella and popliteal webbing E Shortened forearm and hypoplastic radius

Answer A Bicoronal synostosis and maxillary hypoplasia Explanation The patient has clinical findings consistent with a mitten-hand deformity, often associated with Apert syndrome, one of several craniosynostosis syndromes. Bicoronal synostosis is associated with maxillary hypoplasia. Midfacial hypoplasia and shallow orbits cause the eyes to protrude, often associated with hypertelorism, strabismus, and an antimongoloid slant. The palate is usually highly arched and may be associated with clefting and dental anomalies. Ventriculomegaly and central nervous system malformations of the corpus callosum and limbic structures, gyral abnormalities, and white and gray matter anomalies are common. Acne is often severe during adolescence. Cardiac and/or renal anomalies occur in up to 10% of patients. Many patients have associated mental deficiencies—an IQ < 70 occurs in as many as 50% of patients. Moderate-to-severe language delays are common. Early surgery is recommended if there is evidence of increased intracranial pressure. Prompt surgical intervention is also necessary in those patients with respiratory distress due to associated reduction in the size of the nasopharynx, patency of the choanae, and/or anomalies of the tracheal cartilage. In addition to early neurosurgical intervention, patients also require orthopedic evaluation and treatment to optimize function of the digits of the hands and feet. Shortened forearm and hypoplastic radius can be seen in thrombocytopenia with absent radius (TAR) syndrome, which combines thrombocytopenia with an absent radius but with normal thumbs. It can also be seen in vertebral, anal atresia, cardiac, tracheal, esophageal, renal, and limb (VACTERL) defects and in Holt-Oram syndrome, which combines cardiac malformations such as atrial septal defect (ASD) with heart block, absent radii, and absent thumbs. Aplasia cutis congenita and limb reduction defects are seen in Adams-Oliver syndrome. The aplasia cutis in this condition is large. The most common limb malformation is hypoplastic or absent distal phalanges, but the severity of limb anomalies ranges from minor defects such as an absent nail or broad fingertip to more severe involvement. Limb anomalies are usually asymmetric and more commonly involve the lower extremities. Dysplastic sacrum and multiple hemivertebrae are seen in VATER (vertebral, anal atresia, tracheoesophageal, radius and renal anomalies) as well. The vertebral manifestations include malformed vertebrae (missing vertebrae, hemivertebrae, "butterfly" vertebrae, and vertebral clefts and fusions) and ribs (absent ribs, supernumerary ribs, and rib fusions and splitting), and scoliosis in some cases. Absence of the tailbone, sacral agenesis, may also occur. Dysplastic patella and popliteal webbing are seen in popliteal pterygium syndrome (PPS) and/or Escobar syndrome. In PPS, extensive pterygiums are seen from behind the knee down to the heel, with malformed toenails and webbed toes. Dysplastic patella is only occasionally reported in this condition. Facial and genital anomalies are also seen. Escobar syndrome is also called pterygium syndrome and presents with intrauterine growth restriction (IUGR), widespread pterygiums that result in joint contractures, ptosis, cryptorchidism, patellar dysplasia, and foot deformities.

A 4-year-old boy, who is deaf, presents for his 1st health maintenance examination following his adoption after being removed from his home due to medical neglect. His past medical history is unknown. His adoptive parents comment that his eyes are different colors. Heterochromia is confirmed on physical examination. He is also noted to have lateral displacement of the medial canthi, a broad nasal bridge, and a prominent white forelock. Which of the following findings is most likely to be identified during additional evaluation of this patient? A Cutaneous depigmentation B Chipped/broken discolored (bluish-gray) teeth C Bullous eruptions on the hands and feet D Multiple dysplastic nevi E Transverse grooves/furrows along the proximal nail folds

Answer A Cutaneous depigmentation Explanation The patient has clinical signs and symptoms consistent with Waardenburg syndrome, most often caused by mutations in the PAX3 gene, located on chromosome band 2q35. It is characterized by congenital sensorineural hearing loss, pigment abnormalities of the iris (heterochromia), hair hypopigmentation (white forelock involving the forehead and both medial eyebrows), displacement of the medial canthi (dystopia canthorum), facial asymmetry, hypertrichosis of the medial eyebrows, prominent nasal root, and cutaneous depigmentation. Cutaneous findings include patterned areas of depigmentation with sharply defined, irregular borders and scattered "patchy" hyperpigmented macules on normally pigmented skin. Bullous eruptions on the hands and feet describes cutaneous findings in patients with epidermolysis bullosa. This is a genetic condition resulting in very fragile skin that blisters easily. These blisters can be a response to very minor injury or pressure on the skin. Even rubbing or scratching can cause these bullous eruptions. Chipped/broken discolored (bluish-gray) teeth describes dental findings in some patients with osteogenesis imperfecta (OI). OI is a group of genetic disorders that results in poor bone formation, which leads to multiple bone fractures with mild trauma or even no apparent cause. In severe cases of OI, these multiple fractures can occur in utero. Dysplastic nevi, characterized by a variegated color and irregular or angulated borders, are not associated with Waardenburg syndrome. Such findings can be seen in familial atypical multiple mole-melanoma (FAMMM) syndrome and present with multiple family members developing dysplastic nevi and multiple melanomas. This condition does not present with any physical deformities or hearing loss.

A 6-month-old boy presents for his first evaluation following birth after his mother fled from his abusive father and entered a shelter. Reportedly, the boy's father refused to allow his mother to leave the home even for routine pediatric care. He takes a cow's milk-based infant formula and reportedly feeds well. However, his mother reports a history of frequent vomiting. She also describes two previous episodes consistent with generalized seizure activity. On examination, his weight is below the 3rd percentile. He is not able to roll over and has poor head control. His mother also reports that his wet diapers "always smell musty." Which of the following findings is most likely to be identified upon further evaluation of this patient? A Elevated serum levels of phenylalanine B Elevated serum levels of homogentisic acid C Hyperammonemia D Hyperuricemia E Elevated serum levels of alloisoleucine

Answer A Elevated serum levels of phenylalanine Explanation The patient has developmental delay and failure to thrive as a result of phenylketonuria (PKU). Phenylketonuria is an autosomal recessive disorder characterized by the absence or near absence of phenylalanine hydroxylase, which is necessary to metabolize phenylalanine to tyrosine. As a result, phenylalanine accumulates in the serum and is converted into phenylpyruvate (phenyl ketone), which accumulates in the urine and accounts for a pungent, musty odor. Patients characteristically have blond hair, blue eyes, lightly pigmented skin, and numerous eczematous eruptions. Newborn screening programs allow for prompt identification of infants with elevated levels (≥ 7-10 mg/dL) of phenylalanine, often in conjunction with low levels of tyrosine. Dietary restriction of phenylalanine should begin immediately following identification of phenylketonuria on newborn metabolic screening. Breast milk is relatively low in phenylalanine content. Ideally, infants with PKU should be partially breastfed while supplementing with a measured amount of a phenylalanine-free commercial formula. Dietary restrictions are often challenging for affected children because meats, chicken, many fish, breads, pastas, nuts, cheeses, and other dairy products contain phenylalanine. Dietary foods and soft drinks containing aspartame must also be avoided because aspartame contains phenylalanine and aspartic acid. Hyperammonemia is associated with organic acidemias. Elevated serum levels of alloisoleucine are associated with maple syrup urine disease. Elevated serum levels of homogentisic acid are associated with alkaptonuria. Hyperuricemia is associated with Lesch-Nyhan syndrome.

A 10-year-old boy presents for a sports physical. The patient's father recently died of an aortic dissection at 35 years of age. The patient's older brother has had several weeks of chest pain that was attributed to anxiety due to his father's sudden death. His father has 2 sisters, one of whom had surgery to replace her ascending aorta at 30 years of age. She has 3 children; one of her sons has aortic root dilatation. The patient's paternal grandfather died in his 40s of complications related to an aortic aneurysm. Which of the following is the best initial course of action at this time? A Evaluate for Ghent criteria. B Initiate beta-blocker therapy immediately. C Refer for aortic surgery consultation. D Refer for formal audiology exam. E Instruct patient to refrain from any physical exercise

Answer A Evaluate for Ghent criteria. Explanation The patient most likely has Marfan syndrome or a familial thoracic aortic aneurysm syndrome. The Ghent criteria are the clinical diagnostic criteria for Marfan syndrome. As such, it is appropriate to perform the physical exam with attention to the Ghent criteria. Diagnosis is made if there are any 2 of the following major criteria: Ectopia lentis Aortic dilation or dissection Family history Referral for formal ophthalmology exam is appropriate. Likewise, an echocardiogram is in order, based solely on his family history of aortic dissection. Likewise, the patient's older brother should be examined because of his chest pain. While the etiology may be anxiety related to his father's death, both the patient and his brother have a 50% chance of having aortic dilatation and future dissection. Referring the brother for an echocardiogram is also appropriate during this clinic visit.

You are seeing a 17-year-old boy for a routine physical. His mother reports that he seems more immature than his older brothers were at the same age. He has a diagnosis of attention-deficit/hyperactivity disorder (ADHD) and oppositional defiant disorder (ODD). He has an individualized education program (IEP) and is receiving vocational training through the high school. He is not yet shaving. On physical exam, he is tall with disproportionately long limbs. He has scant axillary and pubic hair. He has bilateral gynecomastia. Vision screening is 20/20 bilaterally. After you confirm the suspected diagnosis, which of the following is the most appropriate evaluation for this patient? A Evaluate for testosterone therapy B Sperm count C Echocardiogram D Ophthalmologic exam E Brain MRI

Answer A Evaluate for testosterone therapy Explanation The patient has features of Klinefelter syndrome (47,XXY). He has tall stature with disproportionate limbs. He has evidence of testosterone deficiency (not shaving, scant axillary and pubic hair). He also has behavioral and mild learning disorders (ADHD, ODD, immaturity) common to young men with Klinefelter syndrome. After confirming the diagnosis with genetic testing, evaluation for testosterone therapy is an appropriate next step. Testosterone replacement can produce adequate sexual maturation, increase muscle mass and strength, masculinize body habitus and hair distribution, improve bone density, and improve overall mood. There is little amelioration of gynecomastia. Men with Klinefelter syndrome have a body habitus reminiscent of Marfan syndrome (tall with disproportionately long limbs). But while an echocardiogram would be appropriate for a patient with Marfan syndrome, it is unlikely to reveal anything in Klinefelter syndrome. Although a similar developmental profile can be seen in Fragile X syndrome, which can be associated with mitral valve prolapse, this patient's pubertal anomalies are not consistent with Fragile X.

A 15-year-old male presents to the emergency department with altered mental status and hemiparesis. A CT scan reveals an infarct consistent with a thromboembolic event. Review of his medical records reveals that at birth, he had an abnormal newborn screen. Which of the following disorders was most likely identified on a newborn screen in this patient? A Homocystinuria B Galactosemia C Maple syrup urine disease D Phenylketonuria E Tyrosinemia

Answer A Homocystinuria Explanation Homocystinuria is an autosomal recessive disease caused by a deficiency of cystathionine β-synthase, leading to accumulation of homocysteine in the blood. Excess homocysteine is reconverted into methionine; elevated blood levels of methionine may be detected during newborn screening tests. Patients are often aggressive with some degree of intellectual impairment. They typically have a marfanoid-like appearance in addition to downward dislocation of the lenses (which leads to severe myopia) and orthopedic abnormalities (generalized osteoporosis, arachnodactyly, pectus excavatum, and genu valgum). Thinning and drying of lightly pigmented skin leads to a fair complexion and malar flush. Associated structural abnormalities of collagen cause damage to connective tissue within the vascular endothelium and an increased risk of vascular thrombi in both large and small vessels. About 50% of affected patients respond to treatment with pyridoxine, which enhances activity of the deficient enzyme. Close monitoring of dietary intake of protein is important in all patients, especially those not responding to pyridoxine therapy. Prevalence of the disorder is estimated at 1/200,000-1/350,000. The other distractors are also picked up on newborn screen but do not present with thromboembolic events. Phenylketonuria, if untreated, presents with intellectual disability, seizure, behavioral issues, developmental delays, and lighter skin than relatives.

A 3-month-old boy presents to the emergency department due to excessive irritability and difficulty feeding. Findings on physical examination include a heart rate of 190 beats/minute associated with a gallop rhythm and bilateral basilar rales. An echocardiogram reveals an echogenic, well-defined, 10-mm mass located in the upper portion of the interventricular septum, which is partially obstructing the left ventricular outflow tract. Based on the above information, what other clinical findings may this child also exhibit? A Infantile spasms B Renal tubular acidosis C Idiopathic hypercalcemia D Bile duct paucity E Congenital cataracts

Answer A Infantile spasms Explanation The patient has findings on echocardiogram consistent with a cardiac rhabdomyoma—an intracavitary or intramural tumor often associated with tuberous sclerosis complex. Up to 70% of infants with tuberous sclerosis have the onset of seizures within the 1st year of life. Infantile spasms, partial motor seizures, and generalized tonic-clonic seizures are most common. Cardiac rhabdomyomas are diagnosed prenatally or soon after birth. They are often multifocal and may be asymptomatic or of sufficient size to restrict blood flow, causing cardiac failure and/or arrhythmias. Cardiac rhabdomyomas are most likely to be symptomatic in infancy and early childhood; thereafter, most usually undergo spontaneous regression by 6 years of age. There is no evidence that these tumors undergo malignant transformation. No treatment is indicated for asymptomatic tumors, particularly when first noted in an older child. Dermatologic findings in tuberous sclerosis include hypomelanotic macules (ash-leaf spots), facial angiofibromas (formerly called adenoma sebaceum), grayish-brown and unevenly thickened plaques in the lumbosacral region (shagreen patches), and periungual and ungual fibromas. In addition to seizures, up to half of all patients are intellectually disabled, some profoundly; many have an autism spectrum disorder and/or attention-deficit/hyperactivity disorder. Intracranial hamartomas (tubers) are not uncommon. Renal angiomyolipomas and renal cysts may occur in up to half of all patients. Additional complications include enamel pitting of the permanent teeth, gingival fibromas, retinal hamartomas, and hamartomatous polyps. Management of tuberous sclerosis is symptomatic and organ-specific, focusing on improving the quality of life. Idiopathic hypercalcemia can be seen in Williams syndrome. This condition is characterized by mild-to-moderate intellectual disability or learning problems, unique personality characteristics, distinctive facial features, supravalvular aortic stenosis, and connective tissue weakness. Congenital cataracts is usually an isolated occurrence with no identifiable cause. Over 15 genes involved in cataract formation have been identified. Systemic associations include metabolic disorders such as galactosemia, Wilson disease, hypocalcemia, and diabetes. Cataracts may be a part of a number of syndromes, the most common being trisomy 21. With trisomy 21, the most common cardiac finding is atrioventricular canal defect. Bile duct paucity can be seen in Alagille syndrome. In bile duct paucity, the bile ducts are narrow, malformed, and reduced in number. Consequently, biliary cholestasis results in jaundice, liver damage, and xanthomas. Other hallmark features of Alagille syndrome are peripheral pulmonary stenosis, distinctive facial features, and butterfly vertebrae. Renal tubular acidosis can present with many genetic conditions. It can be seen with deafness, as in renal tubular acidosis and deafness syndrome, and it can be seen associated with blood cell anomalies as in SLC4A1-associated renal tubular acidosis. Other causes include sickle cell disease and various autoimmune conditions.

A 7-year-old boy with a history of attention-deficit/hyperactivity disorder (ADHD) presents for evaluation because his parents are concerned that he is "overmedicated." He has been taking an extended preparation of methylphenidate (36 mg) for the previous 4 months because of impulsive behavior, outbursts of aggression, and difficulty staying on task. Over the last week he has, at times, been "walking like he is drunk" and had difficulty maintaining his balance and climbing stairs. Also, his parents were recently contacted by his teacher because of "slurred speech and poor handwriting." On physical examination, he is visibly upset, refuses to follow simple commands, and is slurring his words. His gait is ataxic. Additional findings include increased deep tendon reflexes in the lower extremities, bilateral ankle clonus, and hyperpigmentation in the axilla and over the knees and elbows. Which of the following tests is most likely to be abnormal upon further evaluation of this patient? A Levels of plasma very long-chain fatty acids B Levels of serum triglycerides, cholesterol, and phospholipids C Levels of serum ceruloplasmin D Levels of glycosaminoglycans in the urine E Levels of serum creatinine kinase, aspartate aminotransferase, and lactate dehydrogenase

Answer A Levels of plasma very long-chain fatty acids Explanation The patient has a history of academic difficulty, impulsivity, and behavioral outbursts associated with the recent onset of gait abnormalities, slurred speech, and deterioration in fine motor skills. These symptoms, in association with evidence of hyperpigmentation on physical examination, are consistent with adrenoleukodystrophy. Classic adrenoleukodystrophy is a disorder of peroxisomal degradation of fatty acid, leading to accumulation of very long-chain fatty acids (VLCFA) in the central and peripheral nervous system and adrenal glands. Inherited via an X-linked recessive pattern, it usually presents between 5 and 15 years of age. Due to associated cortisol deficiency, which occurs in up to 70% of cases, there is increased production of an adrenocorticotropic hormone (ACTH) precursor (proopiomelanocortin [POMC]), which results in increased melanin synthesis and hyperpigmentation of the knees, elbows, and in the palmar creases, axillae, and gingival borders. Additional signs and symptoms include seizures, impaired auditory discrimination, and spatial orientation and visual disturbances. Symmetric periventricular white matter lesions in the posterior parietal and occipital regions of the central nervous system on CT or MRI are commonly identified in affected patients. In addition to treatment of adrenal insufficiency, patients may benefit from a bone marrow transplant. Decreased levels of ceruloplasmin are associated with Wilson disease. Elevated levels of serum triglycerides, cholesterol, and phospholipids are associated with Type 1 glycogen storage disease. Elevated levels of serum creatinine kinase, aspartate aminotransferase, and lactate dehydrogenase are associated with Pompe disease. Elevated levels of glycosaminoglycans in the urine are associated with mucopolysaccharidoses (MPS).

The parents of a 3-year-old African American boy present to the emergency department with concerns that their son "looks pale." They also report decreased activity, subjective fever, and decreased appetite. On physical examination, his heart rate is 160 beats/minute, and his respiratory rate is 36 breaths/minute. Other than appearing pale and having prominent scleral icterus, the remainder of his physical examination is unremarkable. Laboratory values include a hemoglobin concentration of 4.8 g/dL. Which of the following is most likely to have caused this patient's clinical and laboratory findings? A Primaquine B Lamotrigine C Amoxicillin/clavulanic acid D Valproic acid E Griseofulvin

Answer A Primaquine Explanation This patient has clinical and laboratory evidence of hemolytic anemia. Hemolytic anemia is a complication of glucose 6-phosphate dehydrogenase (G6PD) deficiency, the most common red blood cell enzyme abnormality. Certain agents precipitate hemolysis in G6PD-deficient individuals, including aspirin, nitrofurantoin, chloramphenicol, antimalarials, and vitamin K analogs. This disorder is more common in males, among African Americans, and in individuals of Mediterranean, Asian, and Middle Eastern ethnic groups. The hexose monophosphate shunt is responsible for the metabolism of a portion of the total amount of glucose contained within the red blood cells. The first reaction of the pathway is catalyzed by the enzyme G6PD. After several additional steps, reduced glutathione, which inactivates oxidants that damage red cell proteins, is generated. Therefore, a deficiency of G6PD leads to shortened red cell survival and hemolysis following exposure to oxidant stresses. In many G6PD-deficient African Americans, a self-limited hemolysis, followed by recovery within several weeks, occurs soon after exposure to oxidant stresses. Between exposures, however, there is no evidence of anemia or ongoing hemolysis. Testing for G6PD activity in red blood cells should be deferred for several weeks after an acute hemolytic episode because reticulocytes and younger red blood cells predominate immediately after the episode and contain significantly higher enzyme activity than do older cells. Lamotrigine, amoxicillin/clavulanic acid, valproic acid, and griseofulvin do not precipitate hemolysis in patients with G6PD-deficiency because they are not nitrofurantoin, chloramphenicol, or antimalarial. Board Testing Point Recognize G6PD deficiency, and then recall certain agents that may precipitate hemolysis in patients with this disorder.

Which of the following statements regarding Brushfield spots are true? A They form a ring in the midzone of the iris. B They are tiny yellow spots. C They occur especially in children with brown eyes. D They occur in children with trisomy 13. E They are always abnormal.

Answer A They form a ring in the midzone of the iris. Explanation Brushfield spots are common in children with Down syndrome (trisomy 21), and they are whitish spots in a ringlike configuration at the surface of the iris. They can also occur in up to 25% of "normal" kids, especially those with blue eyes.

A mother brings her 1-month-old male to clinic because he seemed "weak" over the last several days. He doesn't feed as well as normal, and his breathing seems "different." He's had no fever or illness, but he does spit up a little after each feeding. He was born at term without complications to a gravida 1 mother. He went home 2 days after birth and was given a healthy report at his newborn visit when he was a week old. On physical examination, he appears alert but hypotonic and keeps his mouth open. Heart sounds are normal but with a slightly increased rate. Liver is approximately 3-4 cm below the right costal margin. Pulses are normal. CXR shows an enlarged heart. ECG shows increased QRS complexes and shortened PR intervals. Glucose level in office is 87 mg/dL (last bottle was 3-4 hours ago). Which of the following is the most likely diagnosis? A Type II glycogen storage disease (Pompe disease) B Hurler syndrome C Type I glycogen storage disease (von Gierke disease) D Duchenne muscular dystrophy E Spinal muscular atrophy

Answer A Type II glycogen storage disease (Pompe disease) Explanation Pompe disease is inherited as an autosomal recessive disorder causing deficient activity of alpha-glucosidase. This results in accumulation of glycogen in muscle, liver, heart, kidney, smooth muscle, and nerves. It's the only condition in which there is such marked hypotonia and cardiomegaly in an infant who was normal at birth. Infants with Pompe disease present with muscle weakness, hepatomegaly, severe cardiomegaly, and normal mental development. They have normal blood glucose, unlike patients with von Gierke disease. The ECG findings of shortened PR intervals and massive QRS complexes are typical for the disease. Enzyme-replacement therapy is now available as treatment. In the juvenile and adult forms, institute a high-protein diet. Nocturnal ventilatory support may also be necessary, which then improves daytime symptoms. In von Gierke disease, the deficit is of glucose-6-phosphatase, which converts glucose-6-phosphate to glucose in the liver. This patient would present at 3-4 months of age (because of the fewer feeds) with severe and rapid hypoglycemia resulting in seizures. There is also hepatomegaly and failure to thrive. It is inherited as autosomal recessive. Spinal muscular atrophy (SMA) is an inherited autosomal recessive disorder. The infantile form (Type 1) presents within the first 6 months of life with hypotonia and weakness, difficulty feeding, and tongue fasciculations. Most die from respiratory failure by 2 years of age. Hurler syndrome (mucopolysaccharidosis Type I, or MPS I) is a lysosomal storage disease in which there is a defect in the gene coding for α-L-iduronidase on chromosome 4p16.3, which is responsible for catabolism of glycosaminoglycans. These patients appear normal at birth and are typically diagnosed within the first 2 years as the phenotypic characteristics appear over time due to accumulation of storage products. They present with coarsened facial features, large tongue, macrocephaly, corneal clouding, and thickened heart valves. Prognosis depends on cardiac involvement, which can be severe, including early cardiomyopathy and death. Duchenne muscular dystrophy is an X-linked recessive disorder and is caused by a mutation in the dystrophin gene that results in absent or deficient dystrophin protein. These boys present in early childhood with "waddling" gait, frequent falls, calf muscle pseudohypertrophy, and Gowers sign. They are non-ambulatory by their teens. Respiratory weakness corresponds with the motor weakness and is a common cause of death.

A 2-day-old girl is ready to be discharged home from the nursery. When reviewing the discharge instructions, the mother informs the nurse that the infant is having difficulty feeding and has been difficult to arouse over the past 2 hours. You come to evaluate the infant and find that she is lethargic and hypotonic. You obtain the following labs: ABG: pH 7.2 PCO2 26 mmHg HCO3- 12 mEq/L Glucose 70 mg/dL Urine ketones Positive Ammonia 658 (elevated) Which of the following is the most appropriate additional laboratory test to obtain now? A Urine organic acid levels B Urine for reducing substances C Plasma very long-chain fatty acid levels D Serum carnitine level E Leukocyte lysosomal enzyme activities

Answer A Urine organic acid levels Explanation In this infant, the test that would be the most directive and suggest a specific diagnosis is urine organic acids. The infant's presentation is most consistent with an organic acidemia, the most common being methylmalonic acidemia and propionic acidemia. With onset of feeding and protein ingestion, the infant with an organic acidemia produces abnormal acids, leading to metabolic acidosis. These abnormal acids secondarily inhibit the urea cycle, leading to elevated ammonia levels; however, the levels are not to the degree commonly observed in ornithine transcarbamylase (OTC) deficiency, a primary urea cycle disorder. To confirm the diagnosis, one needs to demonstrate the abnormal organic acids in the urine. Lysosomal enzyme activities would be obtained if the presentation were consistent with a lysosomal storage disorder. None of these disorders present with neonatal metabolic acidosis and hyperammonemia, both of which are present in this infant. Very-long-chain fatty acids are utilized to diagnose peroxisomal disorders. Zellweger syndrome, while characterized by neonatal hypotonia, should not demonstrate neonatal metabolic acidosis or hyperammonemia. Serum carnitine level may be abnormally low due to the presence of metabolic acidosis. Further evaluation of suspected organic acidemias could include an acylcarnitine profile to further characterize abnormal organic acids; however, free and total carnitine will not contribute to making a specific diagnosis. Reducing substances are helpful in the evaluation of hypoglycemia and should be positive in galactosemia and hereditary fructose intolerance. A negative urine-reducing substance should be negative in an organic acidemia; however, it would also be negative in urea cycle defects and glycogen storage disorders.

A CT scan of the head in a 2-year-old girl with a history of developmental delay and seizures shows evidence of unilateral intracranial calcification in the occipitoparietal region and underlying atrophy of the ipsilateral cerebral hemisphere. A plain skull radiograph reveals that the intracranial calcifications have a serpentine-like appearance. Which of the following is likely to be identified on physical examination of this patient? A A hypomelanotic macule on the trunk B A unilateral facial vascular birthmark involving the upper face and eyelid C Multiple angiomas located in the peripheral portions of the retina D A roughened raised lesion with an "orange-peel" consistency in the lumbosacral region E Multiple hyperpigmented 2-3 mm macules in the axilla

Answer B A unilateral facial vascular birthmark involving the upper face and eyelid Explanation The patient has radiographic findings consistent with Sturge-Weber syndrome, which is caused by anomalous vascular development during the early stages of cerebral vascularization. CT findings commonly include dilation of the ipsilateral ventricle, calcification, and cerebral atrophy. The vascular malformation on the facial skin (sometimes called a port-wine stain) always involves the upper face and eyelid, and sometimes extends over the lower face, oral mucosa, and trunk. Focal seizures, which typically occur on the side contralateral to the facial nevus, often become refractory to antiseizure medications as patients grow older. The majority of patients have intellectual disabilities and require placement in educational facilities capable of meeting the needs of children with significant learning disabilities. Patients also require frequent examination by an ophthalmologist for evidence of increased intraocular pressure in the eye located on the same side of the face as the nevus. A hypomelanotic macule (ash-leaf spot) and a roughened raised lesion with an "orange-peel" consistency (shagreen patch) are seen in patients with tuberous sclerosis; axillary freckling is a sign of neurofibromatosis Type 1; and retinal angiomas present in patients with von Hippel-Lindau syndrome.

A 3-month-old boy is hospitalized after presenting to the emergency department with a seizure. Findings on physical examination include 3 hypopigmented macules located on the lower back and buttocks. His parents report that his 8-year-old brother, who has a seizure disorder and developmental delay, also has similar skin lesions, present since infancy. Which of the following cutaneous findings is most likely to be identified during additional evaluation of the 8-year-old brother? A Cavernous hemangiomas B Angiofibromas C Pemphigus vulgaris D Ichthyosis E Perioral and mucous membrane lentigines

Answer B Angiofibromas Explanation Both the patient and his brother have cutaneous findings consistent with ash-leaf spots, or hypopigmented macules. When ≥ 3 are present, they are considered a major diagnostic criterion of tuberous sclerosis complex. Tuberous sclerosis is an inherited neurocutaneous disorder characterized by the presence of multiple benign hamartomas of the central nervous system, eyes, heart, lung, liver, kidney, and skin. Cutaneous lesions include elliptical or oval hypopigmented macules (ash-leaf spots); facial angiofibromas (fibroadenomas), usually located on malar regions of the face; a shagreen patch; an elevated, thickened, firm, yellowish-orange connective tissue hamartoma; and brownish fibrous plaques on the forehead. Nontraumatic periungual fibromas often develop during later childhood and adolescence. More serious abnormalities associated with tuberous sclerosis include cardiac rhabdomyoma, renal and hepatic hamartomas, and cortical tubers. Tuberous sclerosis is inherited as an autosomal dominant trait with gene defects located on either chromosome 9q or 16q (TSC1 or TSC2), both of which produce a product (which is dysfunctional in patients with the disorder) responsible for tumor-suppressive activity. Seizures, in particular infantile spasms, are common and often difficult to control. The majority of patients have intellectual disability and moderate-to-severe developmental delay. None of the other answers are found in tuberous sclerosis or in conjunction with the scenario provided in the question. Ichthyosis can be seen in genetic disorders that affect the skin. Infants with this condition are born with very hard, thick skin covering most of their bodies. This skin abnormality can affect the shape of the facial structures and limit the movement of the extremities. It can also restrict chest rise and result in respiratory difficulty. Ichthyosis prevents the normal function of the skin to act as a barrier. Seizures is not a finding. Pemphigus vulgaris is a condition which presents with blisters appearing on otherwise healthy-looking skin. Mucous membranes are frequently affected, with blisters first appearing in the mouth. Cavernous hemangioma is a type of hemangioma (blood vessel malformation) that forms a benign growth. Blood through the cavernous malformation is slow and the walls are weak and can leak, resulting in a hemorrhage. Most cases are congenital and can be disfiguring. Cavernous hemangiomas do not tend to regress, unlike capillary hemangiomas. They can occur anywhere in the body, including the brain. Hemorrhage in the brain can result in stroke-like symptoms and/or seizures, but these individuals do not have the other cutaneous findings seen in tuberous sclerosis. Perioral and mucous membrane lentigenes are found in conditions such as Peutz-Jeghers syndrome. Children with this condition present with small, dark-colored spots on the lips and around and inside the mouth. Perianal lentigines are also common. The spots can also occur on the hands and feet. These lentigines fade with age and most are absent by adulthood. Most people with Peutz-Jeghers syndrome develop multiple polyps in the stomach and intestines during childhood or adolescence. Polyps can cause health problems such as recurrent bowel obstructions, chronic bleeding, and abdominal pain. People with Peutz-Jeghers syndrome have a high risk of developing cancer during their lifetimes.

A 5-year-old boy with trisomy 21 presents for a routine health maintenance visit. Which of the following occurs with increased frequency in patients with this disorder? A Spondylolisthesis B Atlantoaxial subluxation C Scoliosis D Avascular necrosis of the femoral head E Clubfoot

Answer B Atlantoaxial subluxation Explanation Individuals with trisomy 21 (Down syndrome) are at increased risk of atlantoaxial instability. The atlas is the 1st cervical vertebra and is responsible for supporting the skull. It is composed of a ring of bone made up of 2 lateral masses joined at the front and back by the anterior and the posterior arches. The 2nd cervical vertebra, the axis, has a blunt, tooth-like process that projects upward. Also referred to as the "dens," or odontoid process, it provides a type of pivot and collar, which allows the head and atlas to rotate around the dens. The incidence of atlantoaxial instability among individuals with trisomy 21 has been reported to be as high as 20%. Spinal cord injury may result when atlantoaxial instability results in subluxation or dislocation of the 1st and 2nd cervical vertebrae. It is a manifestation of the generalized poor muscle tone and joint laxity common in trisomy 21. In severe cases, the 1st cervical vertebra may slip forward and cause spinal cord compression by the odontoid process or by the arch of the 1st cervical vertebrae.

A 10-year-old boy presents for a sports physical. The patient's father recently died of an aortic dissection at 35 years of age. The patient's older brother has had several weeks of chest pain that was attributed to anxiety due to his father's sudden death. His father has 2 sisters, one who had surgery to replace her ascending aorta at 30 years of age. She has 3 children; one of her sons has aortic root dilatation. The patient's paternal grandfather died in his 40s of complications related to an aortic aneurysm. Based on the family history, which of the following is the most likely pattern of inheritance of this disorder? A Autosomal recessive B Autosomal dominant C X-linked recessive D Mitochondrial E X-linked dominant

Answer B Autosomal dominant Explanation The keys for this family are that multiple generations are affected (vertical transmission), both sexes are affected, and there is male-to-male transmission. Criteria for spotting autosomal dominant inheritance are as follows: Both sexes are equally affected. Both sexes transmit to offspring. Present in all generations, although an individual may be undiagnosed due to mild phenotype, incomplete penetrance, or age-related penetrance (e.g., BRCA mutations present in adulthood at incomplete penetrance) Every affected child has a parent with the disorder. (Any affected individual has a 50% risk of passing on the gene mutation to offspring.) Fathers can transmit to sons. Male-to-male transmission eliminates X-linked and mitochondrial inheritance. Vertical transmission with 3 affected generations eliminates autosomal recessive inheritance. To have an autosomal recessive disorder, you inherit 2 mutated genes, 1 from each parent. These disorders are usually passed on by 2 carriers. Their health is rarely affected, but they have 1 mutated gene (recessive gene) and 1 normal gene (dominant gene) for the condition. With each pregnancy, 2 carriers have a 25% chance of having an unaffected child with 2 normal genes, a 50% chance of having an unaffected child who is also a carrier, and a 25% chance of having an affected child with 2 recessive genes. Board Testing Point Be able to discern the expected pattern of inheritance if given a pedigree or extensive family history.

An 18-month-old girl is referred to cardiology for an echocardiogram due to a recently identified systolic murmur. She is taking medications for chronic rhinitis that has been attributed to seasonal allergies. She has had myringotomy tubes placed due to chronic otitis media and conductive hearing loss. Her parents report that she met her early milestones appropriately, but they feel that her developmental progression is now slower than other children her age. She is not yet walking and has several distinct words. On physical exam, her height and weight plot at the 25th percentile, while her head circumference plots greater than the 97th percentile. The anterior fontanelle has not yet closed and is large. Her lips and cheeks are full. She has mild contractures of her fingers. She has a moderate-size umbilical hernia. Her echocardiogram demonstrates a thick mitral valve with insufficiency. Which of the following is the most likely etiology of this child's features? A Metachromatic leukodystrophy B Hurler syndrome C Smith-Lemli-Opitz syndrome D Rett syndrome E Tay-Sachs syndrome

Answer B Hurler syndrome Explanation This child has classic features of a mucopolysaccharidoses (MPS). Of the choices, Hurler syndrome is the only MPS to fit the patient's symptoms. The classic features of an MPS include coarse facial features, frequent upper respiratory infections, macrocephaly, hernias, and thickened heart valves. Metachromatic leukodystrophy is due to arylsulfatase A deficiency. It is characterized by early normal skills followed by progressive neurologic issues, including developmental regression. While there has been slowing of developmental skills, there has been no loss of skills. In addition, the child presents with macrocephaly and mildly coarse features that are not associated with metachromatic leukodystrophy. Tay-Sachs syndrome is due to a deficiency of β-hexosaminidase A (Hex A) with subsequent accumulation of GM2 gangliosides in the neurons. While characterized by hypotonia in the first 6 months of life, there is associated severe developmental delay and regression. Facial features are not coarse nor is there involvement of the heart valves. Smith-Lemli-Opitz syndrome is due to a deficiency of 7-dehydrocholesterol reductase. Characteristic features include microcephaly, anteverted nares, ptosis, and cleft palate. Rett syndrome is characterized by apparently normal psychomotor development during the first 6-18 months of life, followed by a short period of developmental stagnation, then rapid regression in language and motor skills, followed by long-term stability. During the phase of rapid regression, repetitive, stereotypic hand movements replace purposeful hand use.

A 4-month-old girl with a history of prolonged neonatal jaundice presents for evaluation of a heart murmur. Her most recent laboratory findings included a total bilirubin of 11 mg/dL with a direct fraction of 35%. An echocardiogram showed peripheral pulmonic stenosis associated with a ventricular septal defect. A liver biopsy at 3 months of age revealed evidence of chronic cholestasis and a paucity of interlobular bile ducts. Which of the following musculoskeletal findings is most likely to be identified during further evaluation of this patient? A Lumbosacral scoliosis B Butterfly vertebrae C Hypermobility of the large and small joints D Fused cervical vertebrae E Unilateral absence of the radius

Answer B Butterfly vertebrae Explanation The patient has clinical and pathological findings consistent with Alagille syndrome (arteriohepatic dysplasia), the most common syndrome associated with a paucity of intrahepatic bile ducts. Associated skeletal anomalies are common and include abnormalities of the vertebrae, ribs, and hands; butterfly vertebrae; and shortening of the bones of the forearm and phalanges. Affected individuals often present within the first 6 months of life with cholestatic jaundice, hepatosplenomegaly, and pruritus due to elevated bile acids. Xanthomas associated with elevated levels of cholesterol, triglycerides, and phospholipids are common. Facial features, which may not be easily recognized during infancy, include a broadened forehead, pointed chin, hypertelorism, a long straight nose with broad midnose, and an underdeveloped mandible. The most common cardiovascular finding is peripheral pulmonic stenosis, sometimes associated with an isolated atrial or ventricular septal defect or with tetralogy of Fallot. Some patients also have ocular and renal abnormalities. Many fall well below expected growth parameters. Mild-to-moderate intellectual disability may occur.

As a 3-day-old male is being discharged home, the nurse notes that he does not awaken during his newborn heel stick screening. A sepsis evaluation is initiated. In addition, the following labs are obtained: ABG: pH 7.5; pCO2 30; HCO3 23 Serum glucose: 60 mg/dL Urine: negative ketones; negative reducing substances Ammonia: 1,500 µmol/L Urine orotic acid: low Which of the following is the most likely diagnosis: A Medium-chain acyl-CoA dehydrogenase deficiency B Carbamoyl phosphate synthetase deficiency C Galactosemia D Ornithine transcarbamylase deficiency E Propionic acidemia

Answer B Carbamoyl phosphate synthetase deficiency Explanation This infant has symptoms of a urea cycle disorder (UCD), which is characterized by respiratory alkalosis and hyperammonemia. Of the answer choices, only carbamoyl phosphate synthetase (CPS) deficiency and ornithine transcarbamylase (OTC) deficiency are urea cycle disorders. However, only carbamoyl phosphate synthetase deficiency is correct because urine orotic acid is low in this deficiency. Ornithine transcarbamylase (OTC) deficiency and carbamoyl phosphate synthetase (CPS) deficiency are clinically indistinguishable. By 5 days of age, the infant will have poor feeding, lethargy, respiratory alkalosis, and hypotonia due to hyperammonemia (usually > 1,000 µmol/L). If not treated, there is rapid progression to coma and death. Again, to differentiate between OTC and CPS, test urine orotic acid (high in OTC, normal or low in CPS). The classic symptoms of galactosemia are related to hepatic dysfunction and include hepatomegaly, jaundice, and hypoglycemia. The hypoglycemia responds to glucagon and is associated with elevated urine-reducing substances and ketones. Some patients will have E. coli sepsis leading to metabolic acidosis. Propionic acidemia is characterized by metabolic ketoacidosis with or without hyperammonemia. When present, hyperammonemia is not as striking as observed in urea cycle disorders such as OTC and CPS deficiency. Clinically, there is encephalopathy. Vomiting, bone marrow depression, and cardiomyopathy may also occur. Medium-chain acyl-CoA dehydrogenase (MCAD) deficiency is the most common fatty acid oxidation disorder (FAOD). Presentation is usually in the first 2 years of age rather than in the 1st week of life. The characteristic presentation is fasting-induced lethargy and hypoglycemia. Absence of both urinary-reducing substances and ketones in an infant with hypoglycemia suggests an FAOD. Severe hyperammonemia is not a feature of MCAD.

The parents of a 2-week-old female call 911 at 4 a.m. because they believe she is having a seizure. She has not been feeding well for 2 days. En route to the hospital, her glucose level is 30 and her temperature is 101° F (38.3° C) . In the emergency department, you note that her liver edge is past the umbilicus. She is jaundiced. Urine is positive for reducing substances, ketones, and bilirubin. She has an elevated white blood cell count, and her blood culture is positive for gram-negative rods. Of the following, the most likely substance to be elevated is: A Alloisoleucine B Galactose 1-phosphate C Lipids D Medium-chain fatty acids E Insulin

Answer B Galactose 1-phosphate Explanation This patient has features of galactosemia. Symptoms are related to hepatic dysfunction and include hepatomegaly, jaundice, and hypoglycemia. The hypoglycemia responds to glucagon and is associated with elevated urine-reducing substances and ketones. There is an increased risk of E. coli sepsis as well as the development of cataracts. The presentation of galactosemia and hereditary fructose intolerance (HFI) are similar except for age of onset. Galactosemia typically presents in the 1st week of life while HFI typically presents around 6 months of age (after exposure to fructose in juice or sweetened cereal). HFI is not associated with an increased risk of E. coli sepsis. Hyperlipidemia is associated with glycogen storage disease (GSD). The prototype is GSD Type 1. The characteristic time of presentation is at 3-4 months of age when parents are attempting to have the infant sleep through the night (increased length of fasting). Clinically, there is hepatomegaly, failure to thrive, and hypoglycemic seizures. Hypoglycemia is associated with urine ketones but absent reducing substances. There is no response to glucagon. In addition to hypoglycemia, there is lactic acidosis, hyperuricemia, and hyperlipidemia. Liver transaminases are characteristically normal. Medium-chain fatty acids are elevated in medium-chain acyl-CoA dehydrogenase deficiency, the most common fatty acid oxidation disorder. The absence of both urinary-reducing substances and ketones in an infant with hepatomegaly suggests a defect in fatty acid metabolism. Insulin levels are elevated in patients with an insulinoma. Insulinoma presents in the neonatal period with severe hypoglycemia. Elevated insulin levels are also seen in type V hyperlipoproteinemia in which increased triglycerides, chylomicrons, and VLDL are classic. Alloisoleucine is diagnostic for maple syrup urine disease. Infants are well at birth but start having symptoms by 3-5 days of life with rapid progression to death in 2-4 weeks without treatment. Infants commonly have feeding difficulty with poor weight gain, irregular respirations, and seizures.

A 2-year-old girl with a history of intrauterine growth restriction at birth, failure to thrive, and developmental delay presents for evaluation. Her height, weight, and head circumference are all below the 3rd percentile. Facial features include a low hairline; long, thin eyelashes; fine, confluent eyebrows; a long philtrum associated with a downturned upper lip; and micrognathia. Her hands are small, with a hypoplastic proximally shaped thumb, short 5th finger, and mild clinodactyly. Which of the following is most likely to be identified during additional evaluation? A Sensorineural deafness B Hirsutism C Pulmonary stenosis D Coarctation of the aorta E Low-set dysplastic ears

Answer B Hirsutism Explanation The patient has historical and clinical findings consistent with Cornelia de Lange syndrome, a distinctive pattern of anomalies most often the result of a sporadic new autosomal dominant mutation. Characteristic facial features include long eyelashes; thin, "brushed-on" eyebrows that are often confluent (a "unibrow" or synophrys); a shortened nose associated with a long, thin philtrum; and a downturned upper lip, often described as resembling a "cupid's bow." Hirsutism is generalized and distinctive. As described in the vignette, anomalies of the hand are common. Limbs may be hypoplastic with associated contractures. Phocomelia may also occur. Genital anomalies include cryptorchidism and hypospadias in males and a bicornuate uterus in females. Cognitive impairment varies from moderate to severe. In addition to Cornelia de Lange syndrome, a low-set hairline may also be identified in individuals with Turner syndrome and Noonan syndrome. Patients with Turner syndrome have an increased risk of coarctation of the aorta; those with Noonan syndrome have an increased incidence of pulmonary stenosis.

A 5-year-old patient with a known metabolic disorder is started on 500 mg/day of pyridoxine. In order to assess response to treatment, this patient should subsequently have which of the following measured in the serum? A Glycine B Homocysteine C Leucine D Isoleucine E Lysine

Answer B Homocysteine Explanation Inherited in an autosomal recessive manner, homocystinuria is caused by a defect in the gene coding for cystathionine β-synthase, which converts homocysteine into cystathionine during metabolism of dietary protein. The gene defect leads to the accumulation of homocysteine, some of which is reconverted into methionine. Elevated levels of homocysteine increase the risk of vascular disease and thrombus formation, likely due to a combination of endothelial cell damage, proliferation of smooth muscle cells, upregulation of prothrombotic factors, and downregulation of antithrombotic factors. Some patients will respond to pyridoxine (200-1,000 mg/day), which enhances activity of the deficient enzyme. The addition of folic acid and vitamin B12 is recommended in patients who initially do not improve because folate depletion may cause the patient to be unresponsive to pyridoxine. Aspirin is often also recommended for secondary stroke prophylaxis. Measuring homocysteine levels can be used to monitor the effectiveness of treatment. Close monitoring of dietary intake of protein is important in all patients, especially those not responding to pyridoxine therapy. Homocystinuria is associated with various clinical findings including dislocation (subluxation) of the lens in a downward direction. This disorder is commonly characterized by a marfanoid appearance, dry and lightly pigmented skin, coarse hair, pectus deformities, scoliosis, arachnodactyly, genu valgum, and generalized muscular hypotonia. Aortic dilation and/or mitral valve prolapse, common complications in Marfan syndrome, do not occur in homocystinuria. Many patients are mildly intellectually impaired and exhibit aggressive behavior. Elevated leucine is seen in maple syrup urine disease along with isoleucine and valine in both plasma and urine. Finding alloisoleucine is diagnostic of this condition. Dietary restriction of valine, leucine, and isoleucine is the therapy. Glycine elevations are seen in nonketotic hyperglycinemia with the highest level of glycine being in the cerebrospinal fluid (CSF). Sodium benzoate and dextromethorphan have been used for treatment. Isoleucine elevations are seen in maple syrup urine disease, mentioned prior. The catabolic pathway of lysine as well as tryptophan and hydroxyleucine is disrupted in glutaric aciduria Type 1. However, increased amounts of glutaric and 3-hydroxyglutaric aciduria are seen in the urine. Treatment is with L-carnitine, riboflavin, and a special diet.

A 3-year-old boy is noted to have coarse facial features, a widened nasal bridge, and a flattened midface. Hepatosplenomegaly and a large umbilical hernia are identified on abdominal examination. Which of the following radiographic findings would be consistent with the diagnosis of Hurler syndrome in this patient? A Sacral agenesis B Pelvic and calcaneal apophysitis C Dysostosis multiplex in the lower thoracic and upper lumbar vertebrae D Defects of the pars interarticularis in the lumbar vertebrae E Fibrous dysplasia of the long bones

Answer C Dysostosis multiplex in the lower thoracic and upper lumbar vertebrae Explanation Hurler syndrome (MPS 1H) is the most severe subtype of mucopolysaccharidosis Type 1. The mucopolysaccharidoses are lysosomal storage diseases caused by deficiencies of enzymes responsible for the breakdown of mucopolysaccharides (also known as glycosaminoglycans). Partial breakdown products accumulate in the lysosomes, leading to cellular dysfunction and multisystem disorders. Although infants with Hurler syndrome appear normal at birth, coarse facial features become apparent during the 1st year of life. A wide nasal bridge is typically associated with a flattened midface. Recurrent upper and lower respiratory tract infections associated with profuse thickened secretions are common. Many patients have both conductive and sensorineural hearing loss. Enlargement of the tonsils, adenoids, and surrounding soft tissue may progress to partial airway obstruction and cor pulmonale. Hepatosplenomegaly, umbilical and/or inguinal hernias, corneal clouding (which may lead to loss of vision), and cardiomegaly (often associated with signs of heart failure) are often noted on physical examination. Developmental milestones are met only during the first 24-30 months of life and then rapidly decline. Typical cartilage and bony defects, known as dysostosis multiplex, may include beaking of 1 or more vertebral bodies due to hypoplasia of the anterosuperior portion of the vertebrae; proximal pointing of the metacarpals; shortened, thickened clavicles; rounded iliac wings; inferiorly tapered ilia; hypoplastic epiphyses and expansion of the sella turcica; and a thickened diploic space on radiographs of the skull. The average age of death for patients with Hurler syndrome is 5 years of age. It is rare to live beyond 10 years of age. Fibrous dysplasia of the long bones is associated with McCune-Albright syndrome. Apophysitis is common in overuse injuries. Defects in the pars interarticularis in the lumbar vertebrae are often associated with lumbar spondylolysis. Sacral agenesis is seen in infants born to mothers with maternal diabetes. Board Testing Point Recall common radiographic findings in patients with Hurler syndrome.

You receive a consultative report from a neurologist at the regional university medical center for a 7-month-old boy who was referred from your clinic. The child had developed decreased activity with feeding, diminished head control, and increasing irritability. After an initial hospitalization and evaluation, no etiology was detected, and the patient underwent further observation. Over the next several weeks, the patient seemed to have increasing weakness and loss of some developmental milestones. At that point, referral was made for a more in-depth assessment. The results of the consultation reveal that the child has been diagnosed with Leigh syndrome and has been found to have a mitochondrial-based defect in the production of pyruvate dehydrogenase. Which of the following is the pattern of genetic transmission in this situation? A Mosaic pattern B Maternal-to-child transmission C Autosomal recessive D Sex-linked dominant E Autosomal dominant

Answer B Maternal-to-child transmission Explanation Mitochondrial DNA metabolism is independent of nuclear DNA replication. The mitochondria are contributed to the embryo from the mother, and "father-to-child" transmission is not possible.

A 9-year-old boy presents with difficulty releasing his grip on objects. He has also been noted to be tripping over his feet recently and has fallen several times. He lives with his mother. His father died at 22 years of age of a "heart attack." On examination, the patient appears somewhat gaunt for his age; he has moderate temporal wasting and some difficulty enunciating words. The patient has fair strength of the hands but seems unable to easily release his grasp. His proximal muscles appear to have normal strength. He has difficulty with plantar dorsiflexion. His Romberg exam is normal. Which of the following is most consistent with this presentation? A Becker muscular dystrophy B Myotonic dystrophy C Duchenne muscular dystrophy D Charcot-Marie-Tooth disease E Spinal muscular atrophy

Answer B Myotonic dystrophy Explanation Myotonic dystrophy comprises 2 variants, and presentations may vary between individuals. Affected patients typically have involvement of facial muscles with weakness and atrophy. Neck muscles and distal extremity muscles are also affected, with relative sparing of proximal muscle groups. Creatine phosphokinase (CPK) levels are usually normal or mildly elevated. It is an autosomal dominant condition, and new mutations are quite uncommon. Cardiac involvement is common, and conduction defects and sudden death occur in some individuals.

There are 9 essential amino acids that are not synthesized adequately and that humans require in their diets. However, 2 of these can be substituted by other amino acids. Which of the following is a correct substitution pair? A Lysine by tyrosine B Phenylalanine by tyrosine C Histidine by tyrosine D Isoleucine by tyrosine E Valine by tyrosine

Answer B Phenylalanine by tyrosine Explanation The 9 essential amino acids are: leucine, isoleucine, lysine, methionine, phenylalanine, threonine, tryptophan, valine, and histidine. The correct substitution pair is phenylalanine by tyrosine. This substitution occurs in patients with phenylketonuria (PKU), which is an AR disorder in which phenylalanine cannot be converted to tyrosine. Therapy for PKU is limiting the dietary intake of phenylalanine, although to be able to develop and grow normally, PKU patients do need to have some phenylalanine. Another possible substitution occurs between methionine and cysteine, which are the only 2 sulfur-containing amino acids.

An intellectually disabled 8-year-old, blind due to congenital aniridia, presents for a health maintenance examination. At birth, chromosome studies, ordered as part of an evaluation for ambiguous genitalia, confirmed a male karyotype. Which of the following has most likely been periodically performed on this patient in order to screen for a known complication associated with his clinical history and current physical findings? A Hearing assessment B Renal ultrasonography C Fasting lipid panel and hemoglobin A1c D Thyroid function testing E Echocardiogram

Answer B Renal ultrasonography Explanation The patient has a clinical history and physical findings consistent with WAGR syndrome, which is an acronym for Wilms tumor, aniridia, genitourinary malformations, and reduced intellectual disability. Because Wilms tumor is reported in up to 50% of cases, periodic screening with renal ultrasonography is required. Children with this syndrome have a constitutional chromosomal deletion of the WT1 gene, responsible for a transcription factor involved in both gonadal and renal development, located at 11p13. Some patients will develop chronic renal disease, including end-stage renal disease requiring renal transplantation. Wilms tumor is the most common renal malignancy and the 4th most common childhood neoplasm. The most common clinical presentation of Wilms tumor is that of a unilateral, smooth, firm, nontender, and eccentrically located abdominal mass with or without other signs or symptoms, which may include abdominal pain, hematuria (often following incidental and otherwise mild trauma), fever, and hypertension. The lung is the most common metastatic site. Following surgery, chemotherapy, and/or radiation therapy, 5-year survival rates are, on average, 90%. Beckwith-Wiedemann syndrome and Denys-Drash syndrome are also at increased risk for Wilms tumor. Hearing deficits, cardiac lesions, thyroid dysfunction, dyslipidemia, and diabetes mellitus are not increased in WAGR syndrome. Periodic hearing assessment is important in Turner syndrome, as sensorineural hearing deficits are common. XX gonadal dysgenesis that is associated with sensorineural hearing loss is known as Perrault syndrome. Echocardiogram should also be done periodically in Turner syndrome due to the risk of aortic dissection. Periodic thyroid function testing should be done with Turner syndrome, as antithyroid antibodies occur in > 33% of girls with Turner syndrome. The prevalence increases with age. Fasting lipid panel and hemoglobin A1c should be monitored in Turner syndrome and polycystic ovarian syndrome because of increased risk of insulin resistance and a predisposition to Type 2 DM and cardiovascular disease. Board Testing Point Recognize an increased risk of a Wilms tumor and the need for periodic screening with renal ultrasonography.

Patau syndrome findings

Answer B Several sharply demarcated, ulcerated plaques in the absence of overlying skin on the scalp Explanation The patient has sonographic and clinical findings consistent with trisomy 13 (Patau syndrome). Many features of this syndrome are the result of an early defect in the development of the prechordal mesoderm—the origin of the midface, eyes, and forebrain. Other clinical findings may include a sloping, flattened forehead; omphalocele; rocker bottom feet; polydactyly; and congenital heart defects. Aplasia cutis congenita is characterized by an ulcerated, noninflammatory, well-demarcated congenital absence of skin that is usually limited to a small, localized area. Aplasia cutis congenita occurs with increased frequency among patients with trisomy 13. The epidermis, dermis, and, in some cases, the subcutaneous tissues are absent. This disorder is usually limited to the scalp near the sagittal suture or adjacent parietal areas, although occasionally the trunk or extremities are involved. The majority of patients have a solitary lesion and no associated anomalies. Triplet lesions are often seen in association with Patau syndrome. Healing typically occurs over a period of several months as the lesions reepithelialize and are replaced with smooth, grayish, hairless scar tissue. Larger lesions may require surgical intervention. Once healing is complete, most scars are inconspicuous and require no cosmetic intervention. Common clinical findings in trisomy 13 include (think midline defects!): Orofacial cleft (often midline cleft lip; top image) Microphthalmia Postaxial polydactyly of the limbs (bottom image) Holoprosencephaly (cyclopia to premaxillary agenesis) Heart malformations (80%) Hypoplastic or absent ribs Genital anomalies Abdominal wall defects Aplasia cutis congenita Rocker-bottom feet Clenched hands A hairless, solitary, linear, slightly

A 14-year-old intellectually disabled boy recently diagnosed with Marfan syndrome presents to the emergency department after being found on the bathroom floor at school. When evaluated in the emergency department, it becomes clear that he has had a stroke. He is admitted to the intensive care unit. Which of the following findings would provide the best evidence for the actual diagnosis and that the diagnosis of Marfan syndrome was incorrect? A Absence of the corpus callosum on head CT B Elevated levels of serum ammonia C Elevated serum homocysteine D Polyostotic fibrous dysplasia E Butterfly vertebrae

Answer C Elevated serum homocysteine Explanation Patients with homocystinuria have physical characteristics similar to Marfan syndrome, including long thin extremities, arachnodactyly, pectus excavatum, scoliosis, and myopia with or without lens dislocation in an inferonasal direction. However, unlike Marfan syndrome, homocystinuria is often associated with generalized osteoporosis, hypotonia, developmental delay, and intellectual disabilities. Affected individuals also have pale hypopigmented thin skin, which easily becomes flushed, and blond, thin, fragile hair. The most serious risk of homocystinuria is vascular thrombosis, which may present as a stroke, myocardial infarction, or pulmonary embolism. Collagen abnormalities within the connective tissue of blood vessels facilitate the formation of vascular thrombi. Homocystinuria is an autosomal recessive disorder, which caused by a deficiency of cystathionine beta-synthase. This leads to an accumulation of homocysteine in the blood, of which some is converted into methionine; therefore, methionine also is elevated in the serum of affected individuals. Approximately 50% of all patients respond to treatment with pyridoxine, which enhances activity of the deficient enzyme. Close monitoring of dietary intake of protein is important in all patients, especially those not responding to pyridoxine therapy. Early identification and treatment through newborn screening significantly lessens the risk of complications.

A 5-day-old boy is admitted to the pediatric ward due to jaundice and fever. He has not been breastfeeding well, and his mother has been supplementing with a standard formula. A sepsis evaluation is obtained and includes CBC, blood culture, urine culture, and CSF culture. Urinalysis demonstrates ketones and reducing substances. On physical exam, his liver edge is below the level of the umbilicus. Careful eye exam reveals a poor red reflex bilaterally. An ophthalmologic consult confirms small bilateral cataracts. Which of the following is the most likely diagnosis? A Fatty acid oxidation disorder B Glycogen storage disease C Hereditary fructose intolerance D Ornithine transcarbamalase deficiency E Galactosemia

Answer E Galactosemia Explanation The patient's presentation is consistent with galactosemia. Laboratory evaluation to distinguish among the causes of hypoglycemia includes measurement of urinary ketones and reducing substances. Positive nonglucose reducing substances are most suggestive of galactosemia but also may be found in hereditary fructose intolerance. Definitive diagnosis requires measurement of the specific enzymatic activity. Galactosemia is associated with liver dysfunction and jaundice, hepatomegaly, E. coli sepsis, cataracts, urinary ketones, and reducing substances. If a lactose/galactose-restricted diet is provided during the first 10 days of life, the neonatal symptoms quickly resolve and the complications of liver failure, sepsis, neonatal death, and intellectual disability can be prevented. Despite adequate treatment from an early age, children with galactosemia remain at increased risk for developmental delays, speech problems (termed "verbal dyspraxia"), and abnormalities of motor function. A female with galactosemia is at increased risk for primary ovarian insufficiency. Hereditary fructose intolerance (HFI) is due to fructose 1-phosphate-aldolase deficiency. Like galactosemia, HFI is associated with hepatomegaly, jaundice, liver dysfunction, urinary ketones, and reducing substances. Sepsis and cataracts are not features of HFI. Proximal renal tubular syndrome may also be seen in HFI. The hallmark feature of HFI is severe hypoglycemia and vomiting after fructose ingestion. Because the mom has been breastfeeding—with only supplemental standard infant formula—it is unlikely that HFI is the cause of her infant's symptoms. Glycogen storage disease (GSD) may present with severe neonatal hypoglycemia; more commonly, untreated infants present at age 3-4 months with hepatomegaly, lactic acidosis, hyperuricemia, hyperlipidemia, and/or hypoglycemic seizures. Sepsis and cataracts are not features of GSD. The presence of urinary ketones in the absence of reducing substances is most suggestive of GSD. The infant in this vignette had both ketones and reducing substances, eliminating this diagnosis. Ornithine transcarbamylase (OTC) deficiency is associated with elevated ammonia levels with protein intake. The presence of hepatomegaly and jaundice does not suggest this diagnosis. Fatty acid oxidation disorders are characterized by hypoglycemia after a period of a fasting or stress. The absence of both urinary reducing substances and ketones in an infant with hepatomegaly suggests a defect in fatty acid metabolism. As the infant had both ketones and reducing substances, a fatty acid oxidation disorder is eliminated.

Soon after birth, a male at 35 weeks of gestation with a cleft palate is noted to have respiratory distress, which is thought to be the result of upper airway obstruction due to micrognathia and obstruction of the posterior pharyngeal space by the tongue. Which of the following conditions is most often associated with these clinical findings? A Alport syndrome B Stickler syndrome C Arthrogryposis D Sjögren syndrome E Fanconi syndrome

Answer B Stickler syndrome Explanation The patient has clinical findings consistent with Pierre Robin sequence, characterized by the triad of micrognathia, glossoptosis, and resultant airway obstruction. A u-shaped or v-shaped cleft palate occurs in as many as 90% of patients with Pierre Robin sequence. Many conditions are associated with this disorder, most notably Stickler syndrome and chromosome 22q11 deletion syndrome. Nearly 20% of patients with Pierre Robin have Stickler syndrome—a group of several genetic disorders affecting connective tissue, specifically collagen. In addition to its common association with Pierre Robin sequence, Stickler syndrome is associated with ophthalmologic (myopia, cataracts, vitreous anomaly, retinal detachment), auditory (sensorineural or conductive hearing loss), and articular (joint hypermobility, early onset osteoarthritis, spondyloepiphyseal dysplasia) abnormalities. Infants with Pierre Robin sequence require treatment to address both airway obstruction and feeding difficulties. Without intervention, respiratory failure and severe malnutrition are inevitable. Affected infants must be maintained in the prone position, allowing the mandible and tongue to fall forward, reducing airway obstruction. If positioning alone fails to improve symptoms, placement of a nasopharyngeal airway is indicated. Persistent airway obstruction requires surgical intervention. Even with successful airway management, feeding difficulties may persist with patients, requiring additional treatment for swallowing dysfunction, aspiration, and gastroesophageal reflux.

Williams Syndrome

Answer B Strong expressive language skills, socially engaging, often perceived as charming Explanation The patient has facial features consistent with Williams syndrome, which is also associated with supravalvular aortic stenosis. Facial features are often described as "elfin-like" in appearance. Affected individuals are often described as very endearing, socially engaged, and extremely polite, with strong expressive language skills. They typically enjoy interacting with others and are unafraid of strangers. Often they show a preference for interacting with adults rather than with their peers. Supravalvular aortic stenosis is associated with the cardiac findings described in the vignette. Stenosis of other arteries, especially the branch pulmonary arteries, is also common, as are renovascular hypertension and idiopathic hypercalcemia in infancy. Antibiotic prophylaxis to prevent bacterial endocarditis is not indicated in patients with supravalvular aortic stenosis, except in patients with a prior history of endocarditis or a recent (previous 6 months) repair. Williams syndrome is caused by deletion of the elastin gene located on chromosome 7q11.23.

A 2-year-old female is brought in by her new parents for her first physical after being adopted. The adoptive parents have little information about the biological family but have paperwork from the orphanage stating that the parents were first cousins. The adoptive parents report that their daughter has poor muscle tone. Her height plots at the 25th percentile, while her head plots at the 95th percentile. She has evidence of mild dystonia. A brain MRI shows evidence of frontal cortical atrophy and increased extraaxial space. A dilated retinal exam reveals healing retinal hemorrhages. Which of the following is this patient most at risk for: A Thromboembolism B Subdural hematoma C Hypoglycemia D Hepatoblastoma E Alopecia

Answer B Subdural hematoma Explanation This patient has features of glutaric acidemia Type 1 (GA1), an autosomal recessive disorder. Subdural hematoma and retinal hemorrhage, which can be mistaken for child abuse, are associated with GA1. Infants usually present with macrocephaly at birth. Development is normal until the child has a febrile illness or metabolic stressor, which leads to hypotonia and dystonia. CT/MRI shows frontal and cortical atrophy at birth with increased extraaxial space, and, after symptoms of dystonia develop, degeneration of the caudate nucleus and putamen. Striatal degeneration occurs in some patients during the first few years of life if they have a metabolic decompensation. Hypoglycemia is a common feature of disorders of fatty acid oxidation or glycogen storage disease as well as endocrine disorders. Neither dystonia nor macrocephaly is a feature of these conditions. Homocystinuria due to cystathionine β-synthase deficiency is an autosomal recessive disorder. Classic symptoms include Marfanoid habitus (tall stature and long limbs), decreased joint mobility, downward lens subluxation or dislocation, and myopia. Untreated, patients have an increased risk in adulthood of thromboembolism in both the arteries and veins. Alopecia, skin rashes, and encephalopathy are characteristic of multiple carboxylase deficiency and biotinidase deficiency. Seizures, hearing loss, and blindness also can be complications of untreated disease. Both deficiencies are inherited in an autosomal recessive fashion. Tyrosinemia type I (hepatorenal tyrosinemia) is an autosomal recessive disorder caused by deficiency of fumarylacetoacetate hydroxylase, the final step in tyrosine metabolism. Symptoms result from accumulation of succinylacetone. Tyrosine is a maker of the disorder but is not the damaging end product. Failure to thrive and hepatomegaly with hepatoblastoma are the most common presentations. Renal tubular acidosis resembling Fanconi syndrome as well as x-ray findings of rickets also are commonly observed.

A 7-year-old boy is transported to the emergency department following a generalized tonic-clonic seizure while in school. Seizure activity is estimated to have lasted 10 minutes. Physical examination is normal with the exception of a postictal state. He has no prior history of seizures. Several months prior to presentation, he began treatment with methylphenidate for attention-deficit/hyperactivity disorder (ADHD) characterized by an inability to focus and poor academic performance. His evaluation in the emergency department includes an MRI of the head, which reveals symmetric periventricular demyelination in the posterior regions of the cerebral white matter. Which of the following represents the most likely cause of this patient's clinical and radiographic findings? A Deficiency of galactose-1-phosphate uridyltransferase B Accumulation of sphingomyelin C Accumulation of very-long-chain fatty acids D Accumulation of sulfated glycosphingolipids E Deficiency of β-galactosidase

Answer C Accumulation of very-long-chain fatty acids Explanation The patient has clinical and radiographic findings consistent with X-linked adrenoleukodystrophy (X-ALD), a peroxisomal disorder of beta oxidation that results in a defect in the degradation of certain fatty acids, leading to accumulation of very-long-chain fatty acids (VLCFAs) in the white matter of the central nervous system. It begins posteriorly and advances to the anterior portions of the cerebral white matter. VLCFAs may also accumulate within the adrenal cortex, leading to adrenal cortical insufficiency. Inherited as an X-linked disorder, the peak incidence of symptoms is 7 years of age. It is rare prior to 3 and after 15 years of age. Initial symptoms may mimic ADHD, including difficulty concentrating, decreased academic performance, and learning disabilities. As the disease progresses, gait disturbances are common, visual deficits develop—often resulting in blindness, and seizures become common, all within a relatively short period of time. Rapid progression to a vegetative state may occur within as little as 2-3 years—many individuals succumb within 5-7 years. Treatment, with the exception of associated symptoms caused by adrenal insufficiency, is symptomatic. Patients are at risk of developing quadriparesis, swallowing dysfunction, aspiration pneumonia, decubitus ulcers, and contractures. Deficiency of galactose-1-phosphate uridyltransferase causes galactosemia. Galactosemia is checked in newborn screening and, if not treated, the patient is symptomatic in infancy. Accumulation of sulfated glycosphingolipids is seen in arylsulfatase A deficiency, which results in metachromatic leukodystrophy. Individuals with metachromatic leukodystrophy do have neurodegeneration and demyelination; however, they have a course that begins in the 1st to 2nd year of life. Accumulation of sphingomyelin is seen in Niemann-Pick disease, Types A or B. These disorders are due to deficiency in the enzyme sphingomyelin. This results in accumulation in the spleen, liver, lungs, bone marrow, and brain. Type A occurs in Ashkenazi Jews and results in profound brain damage, jaundice, and hepatomegaly. Type B involves hepatosplenomegaly that occurs in the preteen years but with no brain involvement. Deficiency of β-galactosidase activity results in GM1 gangliosidosis, a rare lysosomal storage disorder characterized clinically by a wide spectrum of neurovisceral, ophthalmological, and dysmorphic features.

A 3-year-old girl with a history of a seizure disorder is referred for evaluation of developmental delay. Her birth history is unremarkable. Her weight and height are at the 45th percentile; head circumference is at the 35th percentile. She is easily distracted and excitable―sometimes to the point that she will flap her hands while laughing. Although she is comfortable with her parents and interacts well with them, she exhibits little use of verbal language. She has a flattened occiput and keeps her mouth open, frequently drooling and thrusting out her tongue. On neurological exam, there is evidence of increased tone, tremulous movements of the limbs, and ataxia. Upon chromosome analysis, which of the following is most likely to be identified in this patient? A Reciprocal translocation between chromosome 9 and 22 B Trisomy 21 C Deletion of a gene segment on chromosome 15 D Chromosome 21 mosaicism E Absence of 1 X chromosome

Answer C Deletion of a gene segment on chromosome 15 Explanation The patient described in the vignette has signs and symptoms consistent with Angelman syndrome. This syndrome is an example of genetic imprinting caused by deletion or inactivation of genes on the maternally inherited chromosome 15. By contrast, Prader-Willi syndrome is caused by a deletion or inactivation at the same site on the paternally inherited chromosome 15. Imprinting is a phenomenon by which expression of a certain gene is dependent upon the parent of origin. Imprinted genes are expressed only from either the maternal allele or paternal allele. Children with Angelman syndrome are often easily excitable. Their tendency to laugh and smile while often times flapping their hands or exhibiting other tremulous limb movements led to the colloquial term "happy puppet syndrome"; but this old phrase can be offensive and should not be used. Affected children have significant developmental and speech delays. The majority have associated seizure disorders and characteristic facial features, including a flattened occiput, protruding tongue, wide-spaced teeth, and wide mouth. Behavioral difficulties are such that children must be closely monitored, not only to ensure their safety, but also because they may become easily frustrated and at times aggressive.

A 15-year-old girl presents for evaluation of primary amenorrhea. She also has a history of a bicuspid aortic valve, a horseshoe kidney, and sensorineural hearing loss. She takes no daily medications and has no known allergies. Her height is at the 5th percentile and weight is at the 35th percentile. Breast development and pubic hair distribution are both at sexual maturity rating (SMR; a.k.a. Tanner) Stage 2. Which of the following findings is most likely to be identified during additional evaluation? A Genu varum B Coloboma of the iris C Diffusely enlarged nodular thyroid gland D Diffuse lentigines E Kayser-Fleischer rings in the cornea

Answer C Diffusely enlarged nodular thyroid gland Explanation The patient has primary amenorrhea, short stature, and hearing loss associated with cardiac and renal anomalies, all of which are consistent with Turner syndrome. Chronic lymphocytic thyroiditis (a.k.a. Hashimoto thyroiditis, autoimmune thyroiditis) is the most common cause of thyroid disease in children and adolescents and is associated with both trisomy 21 and Turner syndrome. Clinical findings include an enlarged, sometimes nodular, thyroid that is firm and nontender. Many patients are clinically euthyroid, while others have clinical and laboratory evidence of hypothyroidism with elevated levels of thyroid stimulating hormone (TSH). Thyroid antigen autoantibodies, most often thyroid antiperoxidase and antithyroglobulin antibodies, are identified in up to 95% of individuals with lymphocytic thyroiditis. Patients who are indeed hypothyroid require treatment with levothyroxine and must be closely monitored for changes in the status of thyroid function over time. Euthyroid patients also require close follow-up, especially if there is evidence of subclinical hypothyroidism characterized by elevated levels of TSH and normal levels of thyroxine (T4) and free thyroxine (free T4).

You are called to the newborn nursery to evaluate an infant who has a limb anomaly. The infant is vigorous and feeding well. On physical examination, you note shortened forearms bilaterally with absent radii and present thumbs. Which of the following is the most concerning possible complication? A Cleft palate B Hearing loss C Intracranial hemorrhage D Ventricular septal defect E Malignancy

Answer C Intracranial hemorrhage Explanation Absent or hypoplastic radii with present thumbs is found in thrombocytopenia with absent radius (TAR) syndrome. Thrombocytopenia can occur, although transiently, and can be present at birth or develop later on. A possible complication is brain or other organ hemorrhage from low platelet counts (thrombocytopenia), especially if there is any minor trauma. Cleft palate in this setting can be seen in Roberts syndrome. Roberts syndrome is characterized by prenatal growth restriction (ranging from mild to severe), craniofacial findings (including microcephaly and cleft lip and/or palate), and limb malformations. When there are limb malformations, thumb aplasia or hypoplasia can be seen. Other dysmorphisms are also seen, and significant intellectual disability is usually noted.

A 25-day-old female infant is noted to have a hyperdynamic precordium and bounding pulses. A loud, single 2nd heart sound; systolic ejection click; and a systolic ejection murmur along the left sternal border are present. On chest x-ray, the thymic shadow is absent; biventricular cardiac enlargement and prominent pulmonary vascularity are noted. Chromosome analysis is most likely to reveal which of the following abnormalities? A Unbalanced translocation B Pericentric inversion C Microdeletion D Aneuploidy E A chromosome break

Answer C Microdeletion Explanation The patient has cardiac and radiographic findings consistent with truncus arteriosus in association with absent thymic tissue on chest x-ray, all consistent with DiGeorge syndrome, a disorder caused by microdeletions of specific DNA sequences from chromosome 22q11.2. Clinical manifestations include neonatal hypocalcemia due to parathyroid hypoplasia and cardiac malformations affecting the outflow tract (e.g., tetralogy of Fallot, interrupted aortic arch, truncus arteriosus). Micrognathia, low-set ears, short palpebral fissures, upward or downward slanting eyes, a short philtrum, and small mouth are typical. The characteristic findings in DiGeorge syndrome are the result of disturbance of cervical neural crest migration into the derivatives of the 3rd and 4th branchial pouches. Thymic hypoplasia leads to a range of T-cell deficits and an increase in the percentage of B cells. Concentrations of serum immunoglobulins are usually normal. Opportunistic infections are common in individuals with complete thymic hypoplasia due to a decrease in the number of CD3 T cells. velocardiofacial syndrome is a disorder that lies within a spectrum of clinical presentations caused by a microdeletion at the q11.2 band located on the long arm (q) of chromosome 22. Velocardiofacial syndrome includes as part of its phenotypic spectrum DiGeorge syndrome, which is characterized by thymic dysplasia leading to hypocalcemia, hypoparathyroidism, and immune deficiency. Patients with velocardiofacial syndrome frequently experience postnatal growth retardation and often have deformities of the palate, hypernasal speech, conductive hearing loss, cryptorchidism, hyperextensible hands and tapering fingers, and congenital heart defects (tetralogy of Fallot, right aortic arch, ventricular septal defect). Craniofacial features include narrow palpebral fissures, a prominent nose and a long, narrow face associated with micrognathia. The spectrum of disorders associated with a microdeletion at the q11.2 band on the long arm of chromosome 22 is known by the acronym CATCH 22—cardiac defects, abnormal facies, thymic hypoplasia, cleft defects, hypocalcemia, chromosome 22.

A 2-year-old male has recently been adopted. Due to developmental delays, a brain MRI has been ordered. Per protocol, the child has fasted since dinner last night. Upon arrival to radiology at 8 a.m., he is difficult to arouse. While the nurse is starting his IV, the patient has a tonic-clonic seizure. Bedside glucose is 22, and plasma ammonia is 32. There is mild hepatomegaly. A sepsis evaluation is obtained and includes CBC, blood culture, urine culture, and CSF culture. Urinalysis is negative for ketones and reducing substances. Of the following, the most appropriate additional laboratory test to obtain is: A Leukocyte lysosomal enzyme activities B Urine organic acid levels C Plasma acylcarnitine profile D Plasma very long-chain fatty acid levels E Plasma amino acids

Answer C Plasma acylcarnitine profile Explanation This patient has symptoms of a fatty acid oxidation disorder (FAOD). Medium-chain acyl-CoA dehydrogenase (MCAD) deficiency is the most common FAOD with hypoglycemia after a period of fasting or stress. Age of onset varies from birth to adulthood. The absence of both urinary-reducing substances and ketones in a child with hepatomegaly suggests a defect in fatty acid metabolism. An acylcarnitine profile aids in the diagnosis of FAOD. Specifically, MCAD deficiency is associated with elevation of the C8, C8:1, and C10:1 carnitine esters. Plasma amino acids are used to diagnose disorders of amino acid metabolism, such as phenylketonuria (PKU), tyrosinemia, and maple syrup urine disease (MSUD). Both PKU and tyrosinemia are not associated with hypoglycemia. Tyrosinemia is associated with failure to thrive, hepatomegaly, and hepatoblastoma. Classic MSUD presents with CNS disease in early infancy with a sweet (maple syrup) odor to the urine. Infants are well at birth but start having symptoms by 3-5 days of life with rapid progression to death in 2-4 weeks without treatment. Infants commonly have feeding difficulty with poor weight gain, irregular respirations, and seizures. Urine organic acids aid in the diagnosis of organic acidemias, such as isovaleric acidemia (IVA), propionic acidemia (PA), and methylmalonic acidemia (MMA). IVA is associated with encephalopathy, metabolic acidosis with ketosis, and the odor of sweaty feet. PA presents with severe ketoacidosis with or without hyperammonemia. MMA presents with ketoacidosis, hyperammonemia, and thrombocytopenia. Plasma very long-chain fatty acids are used to diagnose peroxisomal disorders. Zellweger syndrome is the prototype of the class of disorders. Onset of symptoms ranges from birth to 2 years of age with loss of skills and a progressive course. There are mild dysmorphic facial features. There is hepatic dysfunction and jaundice over the course of the disease. Leukocyte lysosomal enzyme activities aid in the diagnosis of lysosomal storage disorders (LSD), such as mucopolysaccharidoses (MPS). All infants appear normal at birth given development of the characteristic features over time as the storage product accumulates in the body. The majority of LSDs are chronic disorders without acute life-threatening neonatal episodes.

You are examining a 4-month-old girl in the emergency department for seizure activity. Blood glucose is 20 mg/dL. Urinalysis is positive for ketones and negative for reducing substances. The abdomen is protuberant, with the liver edge below the level of the umbilicus. Based on your preliminary diagnosis, which of the following is the most appropriate means of preventing nighttime hypoglycemia at this time? A Provide nocturnal administration of parenteral nutrition. B Provide small oral feedings every 2 hours. C Provide a nocturnal infusion of glucose by nasogastric feeding. D Administer a dose of subcutaneous glucagon at night. E Add galactose to the final nighttime feeding.

Answer C Provide a nocturnal infusion of glucose by nasogastric feeding. Explanation The patient's presentation is consistent with glycogen storage disease (GSD) Type I. The current recommended treatment for GSD Type I is noctural infusion of glucose by nasogastric feeding until the age of 2-3 years and frequent daytime feeding. When the child is older, raw cornstarch in water is provided to maintain the blood glucose level. Laboratory evaluation to distinguish among the causes of hypoglycemia include measurement of urinary ketones and reducing substances. Positive nonglucose reducing substances are most suggestive of galactosemia but also may be found in hereditary fructose intolerance. The presence of urinary ketones in the absence of reducing substances is most suggestive of GSD and defects of gluconeogenesis. The absence of both urinary reducing substances and ketones in an infant with hepatomegaly suggests a defect in fatty acid metabolism. Further clinical evaluation, including determination of the age of onset, the timing of the hypoglycemia, and family history may be helpful in further distinguishing among the various possibilities. Definitive diagnosis requires measurement of the specific enzymatic activity. GSD Type I is due to glucose-6-phosphatase deficiency (Type Ia) or glucose-6-phosphate translocase deficiency (Type Ib). GSD Type I is characterized by accumulation of glycogen and fat in the liver and kidneys, resulting in hepatomegaly and renomegaly. Some untreated neonates present with severe hypoglycemia; more commonly, untreated infants present at age 3-4 months with hepatomegaly, lactic acidosis, hyperuricemia, hyperlipidemia, and/or hypoglycemic seizures. Because affected individuals cannot produce endogenous glucose, glucose administration is required to prevent hypoglycemia.

You are called to the newborn nursery to evaluate a full-term infant with abnormal facies. The mother is a 24-year-old G1P1 with no prenatal care who presented to the labor unit after 6 hours of contractions with intact membranes and normal vital signs. The male infant was delivered vaginally without complications, but the nurses notice the infant is small with a "low-set jaw." Physical examination reveals an infant who is small for gestational age and has micrognathia, posteriorly rotated and low-set ears, epicanthal folds, a single transverse palmar crease, and syndactyly of the 2nd and 3rd toes. A hazy appearance of the right cornea is notable when evaluating the red reflexes. Muscular tone is decreased, and the infant sucks and swallows poorly with the 1st feeding. Which of the following is true? A The infant has clinical features suggestive of Down syndrome and may have concomitant congenital heart disease, thyroid disease, and intellectual disability. B The infant has clinical features suggestive of Noonan syndrome and may have concomitant sterility, an undescended testis, and normal intelligence. C The infant has clinical features suggestive of Smith-Lemli-Opitz syndrome and may have concomitant hypospadias, congenital heart disease, and death by 18 months of age. D The infant has clinical features suggestive of trisomy 18 and may have concomitant rocker bottom feet, hypoplasia of nail beds, and normal intelligence. E The infant has clinical features suggestive of trisomy 2 and may have concomitant intellectual disability and heart disease.

Answer C The infant has clinical features suggestive of Smith-Lemli-Opitz syndrome and may have concomitant hypospadias, congenital heart disease, and death by 18 months of age. Explanation Smith-Lemli-Opitz syndrome (SLOS) is an autosomal recessive disorder presenting in ∼ 1/20,000 live births. Defective cholesterol biosynthesis with cholesterol deficiency and accumulation of potentially toxic cholesterol precursor molecules is hypothesized as the etiology of defective CNS myelination and cataract formation. Patients present as small-for-gestational-age infants with microcephaly, a prominent occiput and narrow frontal area, eyelid ptosis, epicanthal folds, strabismus, cataracts, low-set or posteriorly rotated ears, broad nasal tip with upturned nares, micrognathia, single transverse palmar creases, syndactyly of 2nd and 3rd toes, hypospadias with cryptorchidism or ambiguous genitalia, clenched hand abnormalities, cleft palate, bifid uvula, structural abnormalities of the CNS with or without seizures, hypoplastic left heart, and multiple gastrointestinal and renal anomalies. Hypotonia eventually may progress to hypertonia with irritability, shrill screaming, feeding problems, and failure to thrive. Prognosis is poor, even with cholesterol therapy, with 80% of infants dying by 18 months of age. Risk for recurrence in a couple with an affected child is 25% due to the classical Mendelian autosomal recessive inheritance pattern. Many of the features of SLOS are shared with trisomy 18. Infants with trisomy 18 may present with overlapping features of SLOS (intrauterine growth restriction, microcephaly, small face and mouth, congenital heart defects). In addition, the infant with trisomy 18 will typically have rocker bottom feet and clenched fist. Long term survivors (more than 3 years of age) with trisomy 18 invariably have profound intellectual disability. While infants with Down syndrome may also have single transverse palmar creases, epicanthal folds, and congenital heart disease (hypoplastic left heart, AV canal defects, VSDs), the phenotype of Down syndrome patients is somewhat different and includes upward slanting eyes, Brushfield spots, a small upturned nose with a saddle bridge, a small mouth with protruding tongue, a short neck with redundant skin folds, clinodactyly of 5th fingers, and a wide space between the 1st and 2nd toes. Hypotonia during infancy is notable in Down syndrome but does not progress to hypertonia with associated irritability as in SLOS. With directive medical care, Down syndrome patients live into adulthood. Noonan syndrome is characterized by many of the features of Turner syndrome: short stature, web neck, low posterior hairline, broad chest with wide-spaced nipples, peripheral edema, and cardiac defects. Pulmonary stenosis is the most commonly observed cardiac defect associated with Noonan syndrome. The disorder is marked by mental deficiency with normal chromosomes. Trisomy 2 is not compatible with life.

A 3-week-old male is brought to clinic because his mother thinks he looks yellow. He has been taking formula well, about 3-4 oz every 3-4 hours. Growth is good, along the 25th percentile. The infant is having the normal number of wet diapers and is stooling, but mother notices that it is grayish. On physical exam, you notice moderate icterus; he has a small 2/6 holosystolic murmur, and a pointed chin with broad forehead. As you get a chest x-ray, you observe butterfly vertebrae at T5. Liver is palpable to 3 cm below the costal margin; spleen is not palpable. Lab studies show elevated total and direct bilirubinemia with mild elevations in the liver enzymes. Abdominal ultrasound is unremarkable; no gallbladder is seen. Which of the following is the most appropriate next management step? A Bone age B Sweat test C Hemoglobin electrophoresis D Liver biopsy E Urine-reducing substances

Answer D Liver biopsy Explanation The physical features of the pointed chin, butterfly vertebrae, and heart murmur in conjunction with jaundice are consistent with Alagille syndrome, which manifests paucity of bile ducts. A liver biopsy is the next diagnostic test before confirming the diagnosis during surgery. He is growing normally, so a sweat test or urine-reducing substances are unlikely to be useful. Hemoglobin electrophoresis would tell you if he had a hemoglobinopathy, but this is not usually associated with congenital defect. A bone age would not be helpful from a diagnostic standpoint, particularly in this age group.

A newborn female fails her hearing screen for the left ear. When discussing the results with the mother, you note that she has one brown iris and the other is brilliant blue with a few areas of brown. The mother is 25 years of age with several patches of gray hair. She reports that premature graying is "a family trait." Her own hearing is intact, but her brother has bilateral sensorineural hearing loss. Which of the following is the most likely diagnosis: A Goldenhar syndrome B Treacher-Collins syndrome C Waardenburg syndrome D Branchio-oto-renal syndrome E Neurofibromatosis Type 2

Answer C Waardenburg syndrome Explanation This patient has hearing loss due to Waardenburg syndrome, an autosomal dominant condition characterized by a white forelock, premature graying, telecanthus, and iris heterochromia. Goldenhar syndrome is variously known as hemifacial microsomia and oculoauriculovertebral spectrum. Characteristic findings include hemifacial microsomia, epibulbar dermoids, and vertebral anomalies. Conductive hearing loss and preauricular and facial tags are common. Occasionally, there are cardiac and renal defects as well. Neurofibromatosis Type 2 is a cancer predisposition syndrome without obvious dysmorphic facial features. Characteristic findings include hearing loss due to vestibular schwannomas that are typically identified in adulthood. Treacher-Collins syndrome is characterized by mandibular hypoplasia, lower lid colobomas with absent lower lid eyelashes, underdeveloped zygomatic bones, hearing loss, cleft palate, and normal hands. Branchio-oto-renal syndrome is characterized by branchial cleft fistulas or cysts, preauricular pits, hearing loss, and renal anomalies.

You are evaluating a newborn female in the nursery. She has a normal birth weight, but her length is < 5th percentile and her head circumference is > 95th percentile. Her arms and legs appear short proximally. She has midfacial hypoplasia and frontal bossing. She has mild hypotonia. Which of the following is the most likely explanation for these findings? A Turner syndrome B Noonan syndrome C Cornelia de Lange syndrome D Achondroplasia E Russell-Silver syndrome

Answer D Achondroplasia Explanation Achondroplasia is the most common chondrodysplasia, occurring in 1/20,000 live births. In addition to disproportionately short stature with proximal (rhizomelic) limb shortening, findings in infancy include macrocephaly, frontal bossing, and midfacial hypoplasia. The cause of Turner syndrome is loss of part or all of the 2nd sex chromosome. Turner syndrome is reported to occur in 1 in 2,500-3,000 liveborn females. The physical phenotype of individuals with Turner syndrome is highly variable and is often normal, with the exception of short stature. Individuals with Turner syndrome should have normal body proportions and normal head circumference. The most common heart defect is coarctation of the aorta and/or bicuspid aortic valve. Horseshoe kidney is the most common renal malformation. Lymphedema in utero is a major feature of Turner syndrome. The residual puffiness of the hands and feet, redundant nuchal skin and webbing, and abnormal fingernail and toenail configuration are direct consequences of the lymphedema. Russell-Silver syndrome is characterized by intrauterine growth restriction accompanied by postnatal growth deficiency. The birth weight of affected individuals is typically ≥ 2 standard deviations below the mean. Affected individuals typically have proportionately short stature, normal head circumference, 5th finger clinodactyly, typical facial features, and limb-length asymmetry. They also may have café-au-lait spots. Cornelia de Lange syndrome is associated with intrauterine growth restriction. All 3 growth parameters are below average. Limb anomalies range from short fingers to absent upper extremities. Characteristic facial features include synophrys, arched eyebrows, long eyelashes, small upturned nose, small widely spaced teeth, and microcephaly. Although birth length in Noonan syndrome is usually normal, final adult height is typically below normal. The facial features of Noonan syndrome include low-set, posteriorly rotated ears with fleshy helices; vivid blue or blue-green irises; and eyes that are often wide-spaced, with epicanthal folds, thick or droopy eyelids, and downward-slanting palpebral fissures. Other findings can include broad or webbed neck and unusual chest shape. Pulmonary valve stenosis, often with dysplasia, is the most common heart defect and is found in 20-50% of individuals.

A 16-month-old female is brought to the emergency department with generalized tonic-clonic seizures. For the past 3 days she has been vomiting and has had several episodes of diarrhea. She has been tolerating some liquids but has refused all solids. This morning, her parents found her unresponsive with tonic-clonic movement of her extremities. Physical examination reveals an unresponsive toddler. She is tachypneic and tachycardic. She is afebrile. The anterior fontanelle is closed. The liver is moderately enlarged and smooth. The remainder of the exam is normal. A complete metabolic panel reveals a normal anion gap acidosis with glucose of 18mg/dL. Her AST and ALT are 3× normal. Her CBC and urinalysis are normal without ketones present. A CT of the head is normal. CSF cell counts and protein are within normal limits. You infuse a dextrose solution and start ceftriaxone. The child's glucose normalizes and the acidosis improves slightly, but improvement in her mental status is very slow. Serum organic acids are ordered, but the results will not be available for 1 week. Which of the following do you consider? A Broader antibiotic coverage B Thiamine administration C Biotin administration D Administration of carnitine E Exchange transfusion

Answer D Administration of carnitine Explanation This patient has medium-chain acyl-CoA dehydrogenase (MCAD) deficiency. MCAD typically presents with a hypoketotic hypoglycemia and a Reye-like illness during the first 3 years of life. Vomiting, lethargy, coma, and circulatory collapse may occur. Patients tend to be asymptomatic until placed under metabolic stress (e.g., prolonged fasting; a particularly severe presentation is often seen after a gastroenteritis, such as with this patient). Plasma and tissue carnitine levels are depleted by the underlying abnormal fat metabolism. Treatment is frequent carbohydrate feeds and carnitine. Although sepsis must always be considered in an acutely ill child, this would not explain the absence of ketones in the context of hypoglycemia. Also, fever and abnormal CBC would be seen. Adding "broader antibiotic coverage" would not be appropriate at this juncture. There is no indication for the administration of biotin or thiamine as treatment for MCAD. Biotin deficiency leads to alopecia, skin rash, and encephalopathy. Thiamine deficiency is associated with symmetric lower extremity paraesthesias (beriberi) and foot and wrist drop. It is also associated with Wernicke encephalopathy: ophthalmoplegia, ataxia, and confusion. Exchange transfusion is used for severe hyperbilirubinia and is not helpful in the treatment of MCAD. The enzyme deficiency would still be present, preventing conversion of medium-chain fatty acids to energy.

A 16-month-old boy is referred for developmental delay. His mother's pregnancy was uncomplicated. Delivery was unremarkable; birth weight was 7 pounds, 6 ounces. His current weight and height are both at the 97th percentile. Head circumference is at the 75th percentile. Facial features appear coarse. His forehead is prominent with associated hypertelorism. The abdomen is protuberant with both the liver and spleen enlarged. A family history reveals that several maternal uncles and a maternal great-uncle have a facial appearance similar to the patient's and also have large heads and protruding abdomens. Which of the following complications is most likely to occur as this patient grows older? A Hepatocellular carcinoma B Acute myelogenous leukemia C Spastic diplegia D Cardiac valvular leaflet dysfunction E Fanconi syndrome

Answer D Cardiac valvular leaflet dysfunction Explanation The patient has Hunter syndrome, or mucopolysaccharidosis (MPS) Type II. This disorder may be distinguished from other types of MPS by its X-linked rather than autosomal recessive pattern of inheritance. The classic form of Hunter syndrome presents with obvious symptoms by 1-4 years of age. Patients often present with short stature, multiple skeletal deformities, joint stiffness, and coarse facial features. Insufficient levels of iduronate-2-sulfatase allow for the accumulation of mucopolysaccharides (or glycosaminoglycans [GAGs]) in the brain, bones, heart, lungs, liver, and spleen. (While growth is stunted later in life, the growth velocity seen in patients with MPS II is actually accelerated in the first few years of life. At 16 months of age, this child would indeed have a normal or increased height and weight for his age.) Communicating hydrocephalus often develops, leading to further neurological deterioration. The neurologic involvement is progressive and profound by the 2nd and 3rd decades of life. GAG accumulation in the cardiac valve leaflets may lead to cardiac dysfunction and death. The myocardium also may thicken and cause coronary artery compromise. Sudden cardiac death is not infrequent. Fanconi syndrome (renal tubular acidosis, phosphate wasting, and aminoaciduria) and hepatocellular carcinoma can be see in tyrosinemia Type I. Spastic diplegia is seen in arginase deficiency. There are certain conditions, including trisomy 21, Diamond-Blackfan anemia, Bloom syndrome, and Kostmann syndrome, that predispose patients to acute myelogenous leukemia. However, Hunter syndrome is not one that does. Board Testing Point Recognize Hunter syndrome and recall complications associated with this disorder.

A 2-year-old boy with a history of prematurity, intrauterine growth delay, developmental delay, and failure to thrive is noted to have prominent hirsutism and oligodactyly on physical examination. Which of the following is often associated with his physical finding? A DiGeorge syndrome B Cri-du-chat syndrome C Prader-Willi syndrome D Cornelia de Lange syndrome E Klinefelter syndrome

Answer D Cornelia de Lange syndrome Explanation Generalized hirsutism is associated with Cornelia de Lange syndrome, characterized by a distinctive pattern of facial and limb anomalies, which most often occur following a sporadic new autosomal dominant mutation. Characteristic facial features include long eyelashes; thin, well-defined, "brushed-on," confluent eyebrows (synophrys); long, curly eyelashes; a depressed nasal bridge associated with anteverted nares; a long, thin philtrum and a downward-turned upper lip; a shortened neck associated with a low-set hairline; micrognathia; hypoplastic, underdeveloped orbital arches; and low-set, posteriorly rotated ears. Limbs may be hypoplastic with associated contractures. The hands and feet are small, with associated abnormalities of the fingers and toes (e.g., oligodactyly, clinodactyly, partial syndactyly). Phocomelia may also occur. Genital anomalies include cryptorchidism and hypospadias in males and small labia majora and bicornuate uterus in females. Cardiac, gastrointestinal, ophthalmologic, and renal anomalies may also occur.

A 6-year-old boy with a history of Wilms tumor and aniridia undergoes additional testing, which reveals significant global developmental delay. Which of the following is most often associated with this clinical history and findings? A Gastroschisis B Tetralogy of Fallot C Clinodactyly and/or syndactyly D Cryptorchidism E Congenital sensorineural hearing loss

Answer D Cryptorchidism Explanation The acronym WAGR (Wilms tumor, aniridia, genitourinary anomalies, and reduced intellectual abilities) is used to describe a constitutional gene deletion syndrome associated with deletion of one copy of chromosome 11p13. Genitourinary anomalies are observed only in males and may include hypoplastic genitalia, hypospadias, cryptorchidism, fusion and ectopia of the kidney, and duplications of the collecting systems. Patients are also at increased risk of gonadoblastoma, abnormalities in virilization, and, in some cases, obesity. Also known as nephroblastoma, Wilms tumor usually presents between 2 and 5 years of age as an abdominal mass, often discovered by a parent during bathing or during a routine physical examination. Although many patients are asymptomatic, others present with abdominal pain, easy satiety, hemihypertrophy, hypertension, and/or hematuria. On palpation, the masses vary in size, are usually smooth and firm, and may cross the midline. Disease is bilateral in < 10% of all cases. These tumors account for about 6% of childhood cancers and is the 2nd most common malignant abdominal tumor in childhood. On presentation, Wilms tumor may be localized to the kidney; have spread to involve adjacent structures; or have metastasized to the lung, liver, bone, and/or brain. Workup should include CBC, liver and renal function testing, imaging studies of the abdomen (ultrasound, CT, MRI), and chest x-ray. Treatment usually includes both surgical resection and chemotherapy. The other answer choices are not seen in WAGR syndrome. Singly, they can be seen in multiple different genetic conditions or in isolated form.

A 3-year-old developmentally delayed and deaf male is referred for macrocephaly. His head circumference is > 97th percentile. Facial features are coarse; additional clinical findings include joint stiffness, hepatosplenomegaly, thickened skin, corneal clouding, and congenital dermal melanocytosis. These findings are due to a deficiency in which of the following? A Cystathionine β-synthase B Pyruvate kinase C Biotinidase D Lysosomal enzymes E Galactokinase

Answer D Lysosomal enzymes Explanation The patient has clinical findings consistent with Hurler syndrome, also known as mucopolysaccharidosis Type 1. One of several mucopolysaccharidoses, it is acquired via an autosomal recessive pattern of inheritance. The more severe form (Type A) typically presents at 2-4 years of age. Multiple skeletal deformities, joint stiffness, and coarse facial features are associated with developmental delay, progressive hearing loss, and, as patients grow older, short stature. Hypertrichosis, thickened skin, congenital dermal melanocytosis, and whitish papules and nodules are typical. Papules are symmetrically distributed between the angles of the scapulae, along the posterior axillary lines, in the pectoral region, and along the lateral aspects of the extremities. The mucopolysaccharidoses, consisting of more than 7 subclassifications, are caused by a deficiency of lysosomal enzymes required for the degradation of mucopolysaccharides or glycosaminoglycans, which then lead to abnormal accumulation of heparan sulfate, dermatan sulfate, or keratan sulfate. These metabolites then accumulate in the brain, cornea, bones, heart, lungs, liver, and spleen. Communicating hydrocephalus often contributes to progressive neurological dysfunction. Glycosaminoglycan typically accumulates in the cardiac valvular leaflets and myocardium, leading to coronary artery compromise, valvular dysfunction, myocardial disease, pulmonary hypertension, and, in some cases, sudden cardiac death. Milder and more protracted forms of the disease usually present later in adolescence and are not associated with intellectual impairment. No cure for lysosomal storage disorders is available. Treatment includes bone marrow transplant (BMT) that has been shown to arrest neurologic deterioration, stabilize cognition, improve metabolic correction and extend survival. Enzyme replace therapy (ERT) is also available but does not significantly cross the blood brain barrier nor alter neurologic outcome. Long term outcomes are improved the early treatment is provided (with either BMT or ERT). Pyruvate kinase deficiency presents mainly in anemia, jaundice, splenomegaly, and cholecystolithiasis. Galactokinase deficiency is marked by an accumulation of galactose and galactitol caused by mutations in the galactokinase 1 (GALK1) gene. Unlike classic galactosemia, galactokinase deficiency does not present with severe manifestations in early infancy. Its major clinical symptom is the development of cataracts during the first weeks or months of life, as a result of the accumulation in the lens of galactitol, a product of an alternative route of galactose utilization. Biotinidase deficiency results in seizures, hypotonia, hearing and vision problems, ataxia, rashes, alopecia, and developmental delays. Treatment with biotin is lifelong. Partial deficiency results in hypotonia, rash, and hair loss but usually only during times of illness, infection, or stress. Cystathionine β-synthase deficiency results in homocystinuria. These individuals have a Marfanoid habitus, pectus excavatum and pectus carinatum, intellectual disability, connective tissue weakness, thrombotic risks, seizures, psychiatric disease, ectopia lentis (downward dislocation in homocystinuria as opposed to upward in Marfan syndrome), and myopia.

A 2-hour-old boy born at term is noted to have generalized hypotonia, decreased muscle mass, and diffuse weakness. Associated findings include ptosis, poor suck, diminished tendon reflexes, and undescended testicles. Over the next several hours of life, his respiratory efforts become increasingly ineffective, and he is placed on a ventilator. Which of the following is most likely associated with this patient? A History of oligohydramnios identified on prenatal ultrasound B Two vessel umbilical cord identified on physical examination C Nonbillious vomiting following initial feedings D Inability to pass a feeding tube when attempting to gavage feed E History of polyhydramnios identified on prenatal ultrasound

Answer E History of polyhydramnios identified on prenatal ultrasound Explanation The patient has clinical signs and symptoms consistent with myotubular myopathy, a maturational arrest of fetal muscle inherited in its most severe form in an X-linked recessive manner from a clinically asymptomatic mother. Levels of creatine kinase are normal, but muscle biopsy is diagnostic, with evidence of small muscle fibers with abnormal nuclei. Pharyngeal weakness causes difficulty swallowing amniotic fluid, leading to polyhydramnios. Soon after birth, feeding difficulties become apparent, again due to generalized muscle weakness and difficulty sucking and swallowing. The majority of infants die within the first several months of life. Those that survive into childhood have severe physical handicaps. Oligohydramnios is seen in conditions of renal inability to produce urine, such as in polycystic kidney disease. Pyloric stenosis is usually associated with nonbilious vomiting but not with hypotonia, weakness, absent reflexes, or poor suck. A two-vessel umbilical cord may be related to underlying renal anomalies. Renal anomalies, if severe, can present with oligohydramnios but usually do not manifest clinically with the picture described in the history. Inability to pass a feeding tube when attempting to gavage feed represents choanal atresia, which can be seen in CHARGE syndrome (coloboma, heart disease, atresia of the choanae, retarded growth and development, genital anomalies, and ear anomalies and/or deafness). Even if one doesn't recognize the exact etiology of this newborn's clinical findings, know that a newborn with hypotonia, poor suck, and weakness is more likely to have had the same clinical picture in utero, which results in polyhydramnios.

A family, who is new to the area, brings their 10-month-old son to establish care. Their child has marked developmental delays, poor growth, and episodic seizures. He was first noticed to have seizures at 4 months of age, and his development diminished from that point. Of significance is his sparse short hair that is very lightly colored and easily broken. Which of the following tests is most likely to be abnormal? A Elevated serum ceruloplasmin B Decreased serum zinc levels C Elevated urinary metanephrines D Decreased serum copper levels E Increased urinary dermatan sulfate

Answer D Decreased serum copper levels Explanation Menkes disease (a.k.a. Menkes kinky hair disease) is an X-linked recessive dysfunction of copper metabolism with impaired absorption and transport. It is characterized by marked intellectual disabilities and collagen abnormalities resulting in skeletal and vascular pathology and abnormal hair. Copper levels are suppressed. Ceruloplasmin levels tend to also be reduced through feedback inhibition of synthesis. The ceruloplasmin levels, however, tend to rise after IV administration of copper to affected individuals. Diminished zinc levels are associated with acrodermatitis. Dermatan sulfate levels are increased in children with mucopolysaccharidoses such as Hunter, Hurler, and Maroteaux-Lamy syndromes. These syndromes can include intellectual disabilities, but their clinical presentation is typically at or after the first year of life. Elevated urinary metanephrines are seen in cases of pheochromocytoma. This can be seen in paraganglioma-pheochromocytoma syndrome. Board Testing Point Recognize clinical features of Menkes (kinky hair) disease.

A 6-day-old girl, born at home with the assistance of a midwife, presents with persistent vomiting. Her mother reports that she is a poor feeder, taking on average 1 ounce of commercial cow's milk-based formula every 3-4 hours. She is more than 10% below birth weight, has jaundice to below the umbilicus, and is lethargic. Bilateral cataracts are noted. Which organism is most likely to cause sepsis in this patient? A Staphylococcus aureus B Group B Streptococcus C Listeria monocytogenes D Escherichia coli E Herpes simplex virus

Answer D Escherichia coli Explanation This infant has clinical findings consistent with galactosemia, an autosomal recessive disorder caused by a deficiency of galactose 1-phosphate uridyltransferase. Infants with galactosemia are at particular risk of neonatal Escherichia coli sepsis. In infants with this disorder, levels of plasma galactose become elevated following its ingestion as lactose. Galactose is also spilled in the urine and may be detected by a positive reaction for reducing substances. Laboratory manifestations of galactosemia include hyperbilirubinemia, disorders of coagulation, recurrent hypoglycemia, and metabolic acidosis. Cataracts, growth failure, and jaundice are common on physical examination. Prompt elimination of dietary galactose is critical not only for survival but to limit additional impact upon hepatic and renal function. Early diagnosis of galactosemia is facilitated by newborn screening that measures levels of the enzyme galactose 1-phosphate uridyltransferase. The optimal age of treatment is within the first few days of life. Galactosemia is a contraindication to breastfeeding. Soy protein-based/lactose-free formulas are recommended for infants with galactosemia. Herpes simplex virus, Listeria monocytogenes, group B Streptococcus, and Staphylococcus aureus are not associated with galactosemia or with a particular metabolic condition.

A 7-year-old boy is noted to have a history of developmental delay associated with frequent temper tantrums, stubborn refusal to follow directions, and, according to his parents and teachers, an "obsessive need to pick at his skin." His weight is well above the 95th percentile. His parents report a voracious appetite in spite of their attempts to limit his caloric intake. Additional clinical findings include small hands and feet and a hypoplastic penis and scrotum. Which of the following complications is often identified during the neonatal period in patients with these clinical signs and symptoms? A Ichthyosis associated with a collodion B Choanal atresia C Retinal coloboma D Feeding difficulty associated with a poor suck E Prolonged conjugated hyperbilirubinemia

Answer D Feeding difficulty associated with a poor suck Explanation The patient has clinical signs and symptoms consistent with Prader-Willi syndrome, caused by absence of expression of the paternally active genes on the long arm of chromosome 15 (either by a deletion at 15q11-13 or a methylation abnormality). Profound neonatal hypotonia is a characteristic finding, often associated with feeding difficulties, including a poor suck and a weak cry. Hypogonadism is also reported and presents in males as cryptorchidism and scrotal hypoplasia and in females as clitoral hypoplasia upon close examination. As hypotonia resolves, affected toddlers and children manifest symptoms of hyperphagia and will often resort to stealing and hoarding food. Mild cognitive impairment, developmental delay, and behavioral problems are common—affected children are often stubborn, have frequent temper outbursts, and exhibit obsessive-compulsive behaviors. Skin-picking behavior is common. Voracious eating habits frequently lead to choking episodes and gastric distention. As individuals with Prader-Willi syndrome reach adolescence, complications of obesity (sleep apnea, cor pulmonale, diabetes mellitus) frequently develop. Comorbidities include hypothalamic-pituitary dysfunction leading to short stature, central obesity, and hypogonadism. Osteoporosis, scoliosis, and lower-limb alignment abnormalities are common.

A 36-hour-old infant is evaluated in the nursery because of craniofacial abnormalities. He has periorbital fullness and a down-turned lower lip. The iris has a stellate pattern. Further workup reveals an elevated calcium level. An echocardiogram reveals supravalvular aortic stenosis. Of the following, this infant will most likely have: A A cat-like cry in infancy B Paucity of the bile ducts C Wilms tumor D Friendly personality as a toddler E Obesity as a toddler

Answer D Friendly personality as a toddler Explanation The patient in this scenario has features of Williams syndrome (deletion 7q11.23). He has classic facial features (periorbital fullness, down-turned lower lip, stellate iris), hypercalcemia, and supravalvular aortic stenosis. Children and adults with Williams syndrome most often have a friendly or "cocktail party" personality. They also can be described as loquacious. Obesity as a toddler is a feature of Prader-Willi syndrome (PWS), characterized by severe hypotonia at birth and failure to thrive in infancy. By 3 years of age, there is resolution of failure to thrive with the emergence of hyperphagia and obesity. Physical features include bitemporal narrowing, almond-shaped eyes, and small hands and feet. Usually, people with PWS do not have major birth defects. PWS is most commonly due to a de novo deletion of the paternal copy of 15q11-13. Wilms tumor is a defining characteristic of WAGR syndrome (Wilms tumor, aniridia, genitourinary malformations, reduced intellectual abilities). It is due to a deletion of 11p13 involving the PAX6 and WT1 genes. A cat-like cry in infancy is a characteristic of infants with a deletion of chromosome 5p. This condition is variably known as 5p deletion or cri-du-chat syndrome. Additional features include low tone, microcephaly, intellectual disability, and cardiac defects in 33% of affected patients. Paucity of the bile ducts is a characteristic finding of Alagille syndrome. This condition is due to a microdeletion of chromosome 20p12. It also is characterized by pulmonary valve stenosis, posterior embryotoxon, butterfly vertebrae, and a triangular-shaped face.

A 6-year-old girl presents with myoclonic seizures. During evaluation, she is also noted to have significant splenomegaly. Laboratory evaluation shows moderate anemia and borderline thrombocytopenia. Her history is remarkable for poor performance in school and for intermittent nosebleeds. Following stabilization, complete physical examination reveals discomfort of the distal femur. Radiologic evaluation demonstrates an "Erlenmeyer flask" deformity but no acute fracture. Which of the following is the most likely presumptive diagnosis based on these findings? A Niemann-Pick disease B Tay-Sachs disease C Alpha-1-antitrypsin deficiency D Gaucher disease E Wilson disease

Answer D Gaucher disease Explanation Gaucher disease is an autosomal recessive lipid storage disease with a wide spectrum of severity. Three major types of Gaucher disease are recognized, with some resulting in rapid deterioration and death early childhood. Others remain asymptomatic into advanced adult age. There is also a perinatal lethal form in which the infant dies within days to weeks. The disease spectrum includes splenomegaly, hepatomegaly, skeletal abnormalities, and blood manifestations. Types 2 and 3 also demonstrate neurological symptoms. Type 2 is typically fatal by 2 years of age; Type 3, which is what this patient has, is more chronic in course. The disease diagnosis is confirmed by measurement of glucocerebrosidase activity in peripheral leukocytes. Tay-Sachs disease presents with an exaggerated startle reflex in infants. Unlike the Moro reflex, this does not diminish with repeated stimuli. Motor skills are progressively lost. Axial hypotonia, extremity hypertonia, and hyperreflexia are common. In > 90% of infants, a macular cherry-red spot occurs. In Niemann-Pick disease, Type A initially causes vomiting, diarrhea, and failure to thrive. Hepatosplenomegaly is prominent. Neurologic problems occur at ~ 5-10 months of age with hypotonia, progressive loss of motor skills, and reduction in spontaneous movements. Of those affected, 50% have macular cherry-red spots. Respiratory failure is a frequent cause of death—usually by 2-3 years of age. Wilson disease is a disorder of copper accumulation in tissue. The main sites of copper accumulation are the liver and brain, so individuals can present with liver disease, neurological disease (e.g., ataxia), dysarthria, and/or neuropsychiatric symptoms. Alpha-1-antitrypsin deficiency results mainly in pulmonary emphysema, especially in those who are smokers. Liver cirrhosis is also common with certain mutations of the gene. None of these other findings present with the combination of myoclonic seizures, splenomegaly, anemia, thrombocytopenia, and have an Erlenmyer flask-like appearance of the femur on x-ray. This combination is classic for Gaucher.

The pregnant mother of a child in your practice recently learned that her mother had a child who died of "probable metabolic disease" at 2 days of age. She does not know details, and medical records on that child are no longer available. The mother asks if her pregnancy can be tested to see if the fetus could be affected with the same disorder. Which of the following is the most accurate statement regarding metabolic disease in the prenatal setting? A Prenatal metabolic screening is widely available. B Poor fetal growth is common in metabolic diseases. C Fetuses affected with metabolic diseases are unlikely to come to term. D Knowing if a previously affected sibling of the mother was male or female helps determine risk in this and subsequent pregnancies. E Level 2 ultrasonography during the 2nd trimester is likely to be helpful in detecting metabolic disease.

Answer D Knowing if a previously affected sibling of the mother was male or female helps determine risk in this and subsequent pregnancies. Explanation This case highlights the importance of accessing as much information as possible regarding family members who die from metabolic causes. Even if a diagnosis cannot be made in a particular instance, medical records may be helpful in offering the expectant couple guidance or in making a diagnosis posthumously. Knowing the sex of the sibling who died will help establish if this is an X-linked disorder or an autosomal recessive disorder which is much less likely to manifest in our patient's baby unless consanguinity is present. > 100 metabolic disorders can be diagnosed prenatally if clinicians have sufficient information available to know which disorder may occur in a family. Most inborn errors of metabolism do not become evident until after birth. Typically, affected fetuses are normally formed and normally grown, and no abnormalities are detectable on prenatal ultrasonography. Usually affected individuals are born at term. With rare exception, maternal serum screening does not detect metabolic abnormalities in the fetus. Although most of the United States has extended newborn screening panels to detect a variety of inborn errors, no such panel exists for routine screening during pregnancy.

During a health maintenance examination, a 6-year-old boy is noted to be below the 3rd percentile for height. His weight is at the 30th percentile. Several dysmorphic features are also identified, including webbing of the neck, downward slanted palpebral fissures, hypertelorism, and micrognathia. He also has prominent cubitus valgus. A murmur is noted, and an echocardiogram reveals evidence of pulmonary valvular stenosis. Which of the following represents the most likely karyotype in this patient? A 47,XXY B 46,XY/47,XY,+21 C 46,XY/47,XXY D A chromosome break at Xq27, the distal part of the long arm of the X chromosome E 46,XY

Answer E 46,XY Explanation The patient has a normal karyotype associated with abnormal facial features and 2 clinical signs often observed in Turner syndrome (45,X; 45,X/46,XX): webbing of the neck and cubitus valgus. These findings are consistent with Noonan syndrome—an autosomal dominant disorder with variable expression associated with a gene mutation on chromosome 12 but a normal karyotype. Individuals with Noonan syndrome often have right-sided cardiac lesions; in contrast, Turner syndrome is often associated with left-sided lesions, including bicuspid aortic valve and coarctation of the aorta. Noonan syndrome, which occurs in both males and females, is associated with some degree of intellectual disability (∼ 25% of patients), whereas girls with Turner syndrome have normal intelligence, usually excelling in verbal skills but often having problems with spatial relationships (and thus have may difficulty with math). Other findings in Noonan syndrome include delayed sexual maturation, primary ovarian insufficiency in females, and cryptorchidism in males. All patients are at increased risk for bleeding diatheses. Noonan syndrome should always be suspected in any male with phenotypic findings consistent with Turner syndrome. Note that a chromosome break at Xq27, the distal part of the long arm of the X chromosome describes findings consistent with fragile X syndrome. 46,XY/47,XXY refers to a less common Klinefelter syndrome mosaic (others are 46,XY/48,XXYY and 45,X/46,XY/47,XXY). Mosaic results show some cells that have a normal chromosome complement and some cells that show an extra X chromosome. Many of these patients present as tall males with gynecomastia and a female body habitus, and secondary sex development is delayed. They usually have azoospermia, small testes, and infertility; however, when they are mosaic, they may have the ability to reproduce. 46,XY/47,XY,+21 refers to mosaic trisomy 21. Individuals with mosaic trisomy 21 can have milder features of classic trisomy 21 but do not present with the features listed above. 47,XXY is classic Klinefelter syndrome as described above. A chromosome break at Xq27, the distal part of the long arm of the X chromosome, is fragile X syndrome. Males are classically affected, but females can also be affected. (Females who are carriers of the full mutation can show most of the same mutations found in males, and females carrying the premutation tend to be more mildly affected.) These individuals have developmental delays, intellectual disability, and learning disability, and they can have ADHD, seizures, and autism. Their features include a long and narrow face, large ears, a prominent forehead, unusually flexible fingers, flat feet, and, in males, enlarged testicles (macroorchidism) after puberty.

A 12-year-old female with a history of a seizure disorder presents to the emergency department following a generalized tonic-clonic seizure estimated to have lasted for 20 minutes. The patient is lethargic but oriented to time, place, and person. She admits that she has not taken her antiseizure medication regularly, because she is "mad about having to take medicine every day." She also has a history of hypertension and scoliosis. Her blood pressure is 160/98. Physical examination is positive for axillary freckling and an abdominal bruit in the area of the left renal artery. Which of the following physical findings is most likely to be identified upon additional examination of this patient? A A hypopigmented skin lesion on the anterior thigh B A "port-wine" facial nevus in the ophthalmic distribution of the trigeminal nerve C Multiple hemangiomas D Numerous oval macular light brownish-tan lesions on the anterior and posterior trunk E A congenital cataract

Answer D Numerous oval macular light brownish-tan lesions on the anterior and posterior trunk Explanation The patient in the vignette has a history of hypertension associated with a bruit over the renal artery, axillary freckling, and scoliosis. These findings are consistent with neurofibromatosis Type 1 (NF1). The etiology of the patient's hypertension is likely renal artery stenosis caused by a neurofibroma in the left renal artery. "Numerous oval macular light brownish-tan lesions on the anterior and posterior trunk" describes café-au-lait spots. Diagnostic criteria for NF1 in prepubertal patients include the presence of 6 or more café-au-lait spots ≥ 5 mm in diameter. The size of the café-au-lait spots in postpubertal individuals must be ≥ 15 mm in diameter. Other diagnostic criteria for NF1 include axillary and/or inguinal freckling, Lisch nodules (hamartomas in the iris), ≥ 2 neurofibromas or ≥ 1 plexiform neurofibroma, osseous lesions of the sphenoid or long bones (often associated with scoliosis), optic gliomas, and a 1st degree relative with NF1. Two or more of these clinical or historical findings must be present in order to make a definitive diagnosis of NF1. Patients with NF1 are also at increased risk of seizures and intracranial tumors.

A 9-month-old boy presents with his temporary foster parents after being removed from his natural parents due to medical neglect. Since arriving in their home less than 24 hours ago, the foster parents have witnessed 2 episodes of tonic-clonic seizure activity. Reportedly, the patient was delivered at home and has received no routine health care or immunizations. His weight, height, and head circumference are all at the 5th percentile. He is able to roll over, cannot sit without assistance, and has difficulty grasping and transferring a toy. Both his hair and skin are light, with associated eczematous changes. Which of the following enzymes is most likely to be absent or markedly decreased in this patient? A Cystathionine β-synthase B Branched-chain α-ketoacid-dehydrogenase complex C Glucocerebrosidase D Phenylalanine hydroxylase E Galactose 1-phosphate-uridyltransferase

Answer D Phenylalanine hydroxylase Explanation The patient has clinical signs and symptoms consistent with phenylketonuria (PKU), caused by absent or greatly reduced activity of phenylalanine hydroxylase, the enzyme responsible for the hydroxylation of dietary phenylalanine to tyrosine. In this disorder, phenylalanine and its metabolites (including phenylacetate and phenyllactate) accumulate in the blood and urine. In complete enzyme deficiency, serum phenylalanine levels are > 20 mg/dL. The body and urine often have a "mousy" or "musty" odor due to increased concentrations of the metabolite phenylacetic acid. Affected individuals have lightly pigmented skin, eczema, blond hair, and blue eyes. Although normal at birth, untreated patients who were fed either breast milk or standard infant formula quickly develop significant delays in their development and are mentally impaired as they grow older. The mainstay of therapy in PKU is dietary restriction of phenylalanine, beginning immediately upon identification of an affected newborn on routine metabolic screening. Cystathionine β-synthase deficiency causes homocystinuria, and patients with this deficiency have a marfanoid habitus, developmental delay, lens dislocation, and an increased risk of thromboembolism in both arteries and veins. Suspect homocystinuria in someone with subluxation/dislocation of the ocular lens. Lenticular subluxation is usually downward and medial. Maple syrup urine disease (MSUD) is caused by branched-chain ketoacid dehydrogenase (BCKD) complex deficiency. The disease presents in infants after 3-5 days of life, and classic symptoms are urine (or hair or skin) that smells like maple syrup, feeding difficulties, irregular respirations, loss of Moro reflex, and seizures. If untreated, it results in death after 2-4 weeks of life. Galactose 1-phosphate-uridyltransferase (GALT) is deficient in patients with galactosemia. Clinically, infants with this deficiency present with a combination of jaundice, hepatosplenomegaly, hypoglycemia, irritability, cataracts, cirrhosis, intellectual disability, vomiting, seizures, lethargy, poor weight gain, vitreous hemorrhage, and ascites. Glucocerebrosidase deficiency is the cause of Gaucher disease. Infants with this disease are usually normal initially; however, by 2-4 months of age, they start having feeding difficulties and failure to thrive. They develop strabismus, difficulty swallowing, and opisthotonic posturing. They also have huge livers and spleens.

You are seeing a 4-year-old girl for her first physical after being adopted from an orphanage in Eastern Europe. The adoptive parents do not have any information about the birth parents or their families. The adoptive parents report that her global developmental delays are because of her prolonged orphanage stay. She is not yet speaking. She has occasional seizures. She is eating a regular diet. On physical exam, she has fair skin and blonde hair. She has mild eczema and has a musty odor. She does not make eye contact. Head size is less than the 5th percentile but has no unusual facial features. Height and weight plot in the 25th percentile. There is no organomegaly. Which of the following is the most appropriate initial diagnostic evaluation? A Urine mucopolysaccharides B Magnetic resonance imaging (MRI) of the brain C Fluorescence in situ hybridization probe for the 7q11.2 locus D Plasma amino acids E Serum ammonia

Answer D Plasma amino acids Explanation She has features of untreated phenylketonuria (PKU). The easiest way to test for PKU is to assess phenylalanine levels in the blood, either by plasma amino acids or state metabolic newborn screening. In the U.S., all states screen for PKU. This patient was born in an area that does not routinely perform newborn metabolic screening. Classic symptoms of untreated PKU include microcephaly, seizures, severe intellectual disability, and behavior problems. The excretion of excessive phenylalanine and its metabolites can create a musty body odor and skin conditions such as eczema. The associated inhibition of tyrosinase is responsible for decreased skin and hair pigmentation. An MRI of the brain would be unlikely to reveal a significant finding that would lead to a specific diagnosis. Although she has microcephaly and seizures, given her other features, an MRI is not indicated. Urine mucopolysaccharides would not be the most appropriate initial diagnostic evaluation because she does not have short stature or organomegaly, which would suggest a lysosomal storage disorder. Fluorescence in situ hybridization (FISH) probe for the 7q11.2 locus would not be the appropriate first diagnostic evaluation. This FISH probe would be appropriate for a child with clinical findings of Williams syndrome, which is due to a deletion of 7q11.2. Classic facial features include periorbital fullness, down-turned lower lip, and stellate iris. Common medical issues include hypercalcemia and supravalvular aortic stenosis. Children and adults with Williams syndrome are loquacious and most often have a friendly or "cocktail party" personality. Although your patient has microcephaly, seizures, and developmental delay, her other features (pale skin, eczema, musty odor), lack of characteristic behaviors, and normal facial features do not warrant this FISH probe. An ammonia level would not be the appropriate first diagnostic evaluation. She has had no history of acute metabolic decompensation and does not have an aversion to protein, as would be seen in urea cycle defects.

A 5-year-old girl with a history of a seizure disorder presents for evaluation of developmental delay. Her birth history is unremarkable. Her weight and height are at the 35th percentile; head circumference is at the 5th percentile. During the majority of her visit, she interacts with her parents using hand gestures, only rarely verbalizing her needs or answering questions. Her parents describe her as easily distractible, often becoming "so excited that she flaps her hands while laughing and jumping up and down." On physical examination, her skin and hair are lightly pigmented. Her head is small with a flattened occipital area. A left esotropia is noted. She keeps her mouth open, frequently drooling and thrusting out her tongue. On neurological exam, there is evidence of increased tone and tremulous movements of the limbs. Her gait is ataxic, with her arms held outward in the air as she walks. Laboratory testing reveals an abnormality on chromosome 15. Which of the following syndromes shares a defect on the same chromosome that is most likely abnormal in this patient? A Apert syndrome B Crouzon syndrome C Cri-du-chat syndrome D Prader-Willi syndrome E Turner syndrome

Answer D Prader-Willi syndrome Explanation A change in the same location on chromosome 15 can cause 2 different syndromes, depending on whether the affected chromosome is inherited from the mother or the father (i.e., genomic imprinting). This girl has clinical signs and symptoms consistent with Angelman syndrome, caused by loss of gene activity on the maternally acquired chromosome 15 in the 15q11-13 region. By contrast, Prader-Willi syndrome is caused by a deletion or inactivation at the same site on the paternally acquired chromosome 15. Children with Angelman syndrome often have no clinical findings at birth. In the first six months, however, head circumference begins to fall in the microcephalic range, and developmental delay is noted. Speech is significantly delayed, with many having a near absence of speech. A wide-based ataxic gait is also seen. These children often become easily excited, often with bursts of laughter in a socially awkward or inappropriate manner while flapping their hands. (The old phrase, "the happy puppet," used to describe children with Angelman syndrome, can be offensive and should not be used.) Angelman syndrome is also associated with characteristic facial features, including microcephaly, hypopigmentation, a flattened occiput, protruding tongue, wide-spaced teeth, and a wide mouth. Many affected individuals become easily frustrated and often become aggressive, especially if they perceive that their needs are not met or they do not get their own way. Apert and Crouzon syndromes are both due to mutations in the FGFR2 gene located on chromosome 10. Cri-du-chat syndrome is due to a deletion of a portion of chromosome 5. Turner syndrome is due to deletion of an X chromosome.

You are asked by a colleague to review the course of a child of Louisiana Cajun descent, who was apparently normal until 6 months of age. Exam at that time was only remarkable for an exaggerated startle response. From that date, she started to show lack of normal development, increasing muscle weakness, and hearing loss. Concerns have recently been raised about her vision as well. On examination at the age of 11 months, the child is unable to sit unsupported. Her exam also shows the development of a cherry-red spot on funduscopic exam. Which of the following is the most likely diagnosis? A Niemann-Pick disease B Wilson disease C Gaucher disease D Tay-Sachs disease E X-linked adrenoleukodystrophy

Answer D Tay-Sachs disease Explanation Tay-Sachs disease is an autosomal recessive defect in ganglioside metabolism that is most prominent in persons of Eastern European Jewish ancestry and in French Canadian and Cajun populations. The accumulation of lipids in neural structures leads to progressive neurological deterioration with loss of vision, stamina, and muscle tone. An early sign is often an increased startle response, usually appearing at 3-6 months of age. Physical exam demonstrates the development of a cherry-red spot on the retina. This occurs because of ganglioside deposition in the retina, leaving the fovea as the normal "island" in an abnormal retina. The diagnosis in these patients is confirmed by decreased serum levels of hexosaminidase A. Niemann-Pick disease is a lysosomal storage disorder characterized by hepatosplenomegaly, ataxia, dysarthria, and dysphagia. Gaucher disease is caused by a deficiency of glucocerebrosidase and is characterized by hepatosplenomegaly, spasticity, and seizures. Wilson disease is characterized by onset of liver dysfunction and neuropsychiatric manifestations, including cognitive dysfunction and parkinsonism. Patients are affected between 6 and 20 years of age. X-linked adrenoleukodystrophy occurs in males and is characterized by developmental regression, spasticity, and no hepatosplenomegaly.

You are seeing a 3-year-old male for the first time. He is very friendly and outgoing. Mentally, he is well below average. He is at the 15th percentile for weight and 5th percentile for height. He has short palpebral fissures with depressed nasal bridge, long philtrum, and prominent lips with open mouth. He has a hoarse voice and a low-pitched, rough, ejection systolic murmur heard best at the sternal edge of the 1st right intercostal space. You also feel a thrill in the suprasternal notch. No webbing of the neck. He also has short nails. Which of the following is the most likely diagnosis? A Noonan syndrome B Kabuki syndrome C Costello syndrome D Williams syndrome E Smith-Lemli-Opitz syndrome

Answer D Williams syndrome Explanation The child being described has Williams syndrome, which is caused by deletion of one elastin allele located on chromosome 7q11.23. These children have short stature, intellectual disabilities, friendly loquacious personality, and hoarse voice. The facies of the patient described is typical, as well as epicanthal folds and blue eyes with stellate pattern in the iris. The murmur described is that of supravalvular aortic stenosis (SVAS), which is commonly found in Williams syndrome, although other heart defects may also be seen. People with Williams syndrome typically have outgoing, engaging personalities and tend to take an extreme interest in other people. They are referred to as having a "cocktail party" personality. Distinctive facial features include a broad forehead, a short nose with a broad tip, full cheeks, and a wide mouth with full lips. The cardiovascular issues usually include SVAS, although other congenital heart defects have been reported. In addition, it is important to remember that individuals with Williams syndrome are at risk for having increased calcium levels.

A newborn female is evaluated in the nursery because of hypotonia, flat midface, epicanthal folds, and Brushfield spots. An echocardiogram reveals an atrioventricular canal defect. The most likely karyotype is: A 46,XX B 47,XX,+18 C 45,X D 47,XXY E 47,XX,+21

Answer E 47,XX,+21 Explanation This patient has Down syndrome. Down syndrome is most commonly due to free trisomy 21 (47,XX,+21). In addition to characteristic facial features and increased nuchal folds, ∼ 50% of those with Down's will have a congenital heart defect; about 1/3 are atrioventricular canal defects, while another 1/3 have a ventricular septal defect (VSD). Peripheral edema in a newborn female is concerning for Turner syndrome. Coarctation of the aorta is the characteristic cardiovascular birth defect for Turner syndrome. The most common karyotype for Turner syndrome is 45,X. These individuals are phenotypically female.

An 11-year-old boy is found to have a widely split and fixed 2nd heart sound associated with a medium-pitched Grade 3/6 murmur, best heard along the left middle and upper sternal border. 1st degree heart block is present on electrocardiogram. Which of the following is most likely to be identified on physical examination of this patient? A Polydactyly B Hemihypertrophy C Talipes equinovarus (clubfoot) D Spina bifida E A bifid right thumb

Answer E A bifid right thumb Explanation The patient has evidence of a conduction delay on electrocardiogram and auscultatory findings consistent with an atrial septal defect (ASD). Unilateral or bilateral asymmetric upper limb (e.g., radius, thumb, carpal bones) abnormalities associated with an ASD or a ventricular septal defect (VSD) are consistent with Holt-Oram (heart-hand) syndrome. The radii and/or thumbs may be absent, hypoplastic, or deformed and the carpal bones fused. The ulna and bones of the lower extremity are unaffected. A spectrum of conduction defects, ranging from 1st degree to complete heart block, commonly complicate the disorder. Inherited via an autosomal dominant pattern, the gene responsible for Holt-Oram syndrome is located on chromosome 12. An ASD, most often located in the region of the fossa ovalis, is usually asymptomatic and discovered only during routine physical examination. The 2nd heart sound is characteristically widely split and fixed in all phases of respiration. A Grade 2/6 to 4/6 systolic murmur, best heard along the mid-to-upper left sternal border and produced by increased flow across the right ventricular outflow tract into the pulmonary artery, may be accompanied by a short middiastolic murmur caused by increased blood flow through the tricuspid valve.

A 9-year-old boy returns for follow-up after elective repair of an inguinal hernia. On physical examination, wound dehiscence and poor healing are noted at the surgical site. His mother reports that similar complications occurred several years ago after a laceration was sutured from a fall while playing at school. The overall texture of his skin on physical examination is unusually smooth with a doughy consistency. Which of the following is often associated with these historical and clinical findings? A Linear streaks of dermal hypoplasia associated with telangiectasia B Microphthalmia associated with a retinal coloboma C Psoriatic plaques overlying the elbows and knees D Multiple joint contractures E A midsystolic click followed by a late systolic murmur heard best at the cardiac apex

Answer E A midsystolic click followed by a late systolic murmur heard best at the cardiac apex Explanation The patient in the vignette has a history of poor wound healing associated with a doughy, smooth texture of the skin, all consistent with Ehlers-Danlos syndrome. This syndrome represents a heterogeneous group of connective tissue diseases characterized by defects in collagen synthesis, which may affect the skin, ligaments, joints, and blood vessels. Mitral valve prolapse, characterized on physical examination by a midsystolic click often followed by a late systolic murmur, is a common manifestation of this disorder. Aortic root dilation/dissection is an additional and potentially far more serious cardiac complication in some patients. Clinical hallmarks of the disorder include joint hypermobility ("double-jointed"), hyperelasticity of the skin, and tissue fragility associated with easy bruisability and poor wound healing. Multiple disease types have been identified. Certain types are associated with increased risk of spontaneous rupture of arteries, particularly midsized arteries.

A 1-day-old male is noted to have difficulty feeding due to micrognathia. A chest x-ray, obtained due to concern about possible aspiration after an episode of choking during attempts to feed, reveals an absent thymus. Soon after the x-ray is obtained, he is noted to have generalized tonic-clonic seizure activity. The patient is subsequently transferred to the neonatal intensive care unit, where additional evaluation includes an electrocardiogram. Which of the following findings is most likely to be identified upon review of the electrocardiogram? A Prolonged PR interval B Peaked T waves C 2nd degree Mobitz 1 heart block (Wenckebach phenomenon) D ST-segment elevation E A prolonged QTc interval

Answer E A prolonged QTc interval Explanation The patient in the vignette has an absent thymus on chest x-ray and neonatal seizures that are likely a result of hypocalcemia due to parathyroid hypoplasia. These findings are consistent with DiGeorge syndrome. Hypocalcemia results in several abnormalities on electrocardiogram, which may include a prolonged QTc interval and prolonged ST interval. T-wave changes are occasionally seen, but far less than prolonged QTc or ST intervals.

The parents of a 3-year-old girl are concerned about their daughter's developmental delay. The parents report that delays were first noted in early infancy. She only recently began to walk with the aid of a reverse walker. She has no speech and communicates with gestures and vocalizations. She is a happy and easily excitable child. On physical exam, her height and weight plot at the 25th percentile; however, her head circumference is less than the 3rd percentile and is at the 50th percentile for a 15-month-old. She ambulates with support but has a wide-based ataxic gait. Which of the following is the most likely etiology of this child's features? A Williams syndrome B Fragile X syndrome C Prader-Willi syndrome D Rett syndrome E Angelman syndrome

Answer E Angelman syndrome Explanation Angelman syndrome is characterized by severe developmental delay, severe speech impairment, gait ataxia and/or tremulousness of the limbs, and a unique behavior with an inappropriate happy demeanor that includes frequent laughing, smiling, and excitability. In addition, microcephaly and seizures are common. It is caused by loss of gene activity on the maternally acquired chromosome 15 (at 15q11-13). Some have referred to individuals with Angelman syndrome as the "happy puppet," referring to the wide-based gait, arms usually raised for balance, and frequent outbursts of laughter; this old phrase can be offensive and should not be used to describe patients. Williams syndrome is associated with calcium disturbances throughout life but primarily during infancy. Retrospectively, most children with elevated calcium levels are noted to have a significant number of colic symptoms. An overly friendly personality without appropriate social boundaries for age is characteristic for Williams syndrome. Typically, people with Williams syndrome are described as loquacious or as having a "cocktail party" personality. Supravalvular aortic stenosis is also seen frequently. Classic facial features of Williams syndrome include puffy eyelids and a full mouth. Fragile X syndrome is the most common inherited form of intellectual disability. Males have a characteristic appearance (large head, long face, prominent forehead and chin, protruding ears), connective tissue findings (joint laxity), and large testes after puberty. Behavioral abnormalities, sometimes including autism spectrum disorder, are common. Females may also be affected but to a lesser degree than males. Prader-Willi syndrome is associated with infantile hypotonia and failure to thrive. By early childhood, there is resolution of failure to thrive with the emergence of hyperphagia. Physical features include bitemporal narrowing, almond-shaped eyes, and small hands and feet. Prader-Willi is due to loss of function of the paternally inherited deletion at 15q11-13. Rett syndrome is characterized by apparently normal psychomotor development during the first 6-18 months of life, followed by a short period of developmental stagnation, then rapid regression in language and motor skills, followed by long-term stability. During the phase of rapid regression, repetitive, stereotypic hand movements replace purposeful hand use. Additional findings include fits of screaming and inconsolable crying, autistic features, panic-like attacks, bruxism, episodic apnea and/or hyperpnea, gait ataxia and apraxia, tremors, seizures, and acquired microcephaly.

A newborn boy is noted to be cyanotic unless he is crying. Several attempts to pass a small feeding tube through the nares are unsuccessful, and a tracheotomy is required. Dysmorphic features include short palpebral fissures, prominent forehead, hypertelorism, and low-set ears. Additional findings include a ventricular septal defect on echocardiogram and a coloboma of the retina. Which of the following is most likely to be associated with this patient? A Thrombocytopenia B Lissencephaly C Arnold-Chiari malformation D Neurofibromas E Cryptorchidism

Answer E Cryptorchidism Explanation The patient has clinical findings of CHARGE syndrome, characterized by coloboma of the retina, heart abnormalities, atresia of the choanae, retarded growth and mental development, genital abnormalities in males, and ear abnormalities, including deafness. Males are often born with an unusually small penis (micropenis) and undescended testes (cryptorchidism). Common cardiac defects include abnormalities of the aortic arch and/or right subclavian artery, ventricular septal defects, and tetralogy of Fallot. Dysmorphic facial features include short palpebral fissures, ptosis, hypertelorism, low-set dysplastic ears, prominent forehead, and "cupid's bow" mouth. Chromosome studies are normal. Because infants are obligate nose breathers, bilateral atresia of the choanae will present clinically with cyanosis that resolves with crying. A small feeding tube cannot be passed through the atretic nares. Tracheostomy is required to relieve upper airway obstruction. Many patients die in early life due to complications of their congenital abnormalities; those who survive are developmentally delayed with a broad range of IQs (30-80) . In order to make the diagnosis of CHARGE syndrome, at least 4 of the 6 categories of abnormalities must be present, at least one of which must be coloboma or choanal atresia. Mutations that result in CHARGE syndrome have been identified in the CDH7 gene.

An 11-year-old boy is brought to the clinic by his mother for complaints of shoulder strain. The boy has been playing tennis since early childhood, but his mother states that recently he has become increasingly unable to serve the ball or complete an overhead volley. He has also complained that his legs get tired much more easily than they did before. On exam, he has normal passive range of motion in the shoulders, but he has weakness when elevating his arms above his head. In the lower extremities, strength is 3-4/5, despite prominent calf muscle size. Laboratory evaluation reveals a CPK of 667 U/L (normal 150-499 U/L). A muscle biopsy reveals markedly reduced levels of dystrophin. Which of the following diagnoses is most consistent with this presentation? A Duchenne muscular dystrophy B Dermatomyositis C Juvenile amyotrophic lateral sclerosis D Myotonic dystrophy E Becker muscular dystrophy

Answer E Becker muscular dystrophy Explanation Becker muscular dystrophy resembles Duchenne muscular dystrophy in many aspects but tends to follow a less severe course. It is an X-linked recessive disease that leads to defective dystrophin production but not a total absence of the protein. The disease primarily affects proximal muscles, and CPK levels are typically elevated. By definition, these patients retain ambulatory abilities after 15 years of age. Life expectancy can be into the 4th or 5th decade. Duchenne muscular dystrophy is an X-linked recessive myopathy that predominantly affects proximal muscle groups. The disease is related to gene defects that lead to an absence of the protein dystrophin rather than a deficiency. Affected individuals are typically wheelchair bound by 12 years of age and die of respiratory failure in their teens to early 20s.

An 8-year-old boy with short stature is noted to have dysmorphic facial features, which include a triangular-shaped face with downslanting palpebral fissures, hypertelorism, and a short, webbed neck. Musculoskeletal findings include joint laxity, cubitus valgus, and pectus deformity. The 1st heart sound is normal and followed by a systolic ejection click, which is louder on inspiration; a systolic ejection murmur is best heard at the 2nd left intercostal space. Review of this patient's past medical history is most likely to reveal which of the following? A Recurrent nephrocalcinosis B Inflammatory bowel disease C Pancreatic insufficiency D Proximal muscle weakness E Bleeding diatheses

Answer E Bleeding diatheses Explanation The patient has clinical findings typical of Noonan syndrome, including a murmur consistent with pulmonic stenosis, which may be caused by mutations in several genes; PTPN11 (50% of cases), SOS1, and RAF1 mutations are the most common. Bleeding diatheses in these patients are common; the most frequent abnormality is Factor 11 deficiency. Combined coagulation disorders and thrombocytopenia may also be identified. Excessive bruising; recurrent epistaxis; prolonged bleeding following injury, childbirth, or surgery; and menorrhagia are common complaints. Other features include short stature, congenital heart defects (most commonly, pulmonary valve stenosis), pectus excavatum, webbed neck, low-set ears, hypertelorism, and lymphedema.

A plain radiograph of the skull of a 4-month-old boy with a cleft palate and dysmorphic facial features reveals hypoplasia of several facial bones, including the malar bones, zygomatic processes, lateral pterygoid plates, and mandibular condyles. The paranasal sinuses are also hypoplastic, and the mastoid bones are not pneumatized. Facial features include malformed pinnae; short, downward-slanting palpebral fissures; micrognathia; and colobomata along the lateral lower lid, which also lacks eyelashes medially from the lower lid. Which of the following is most likely to be a necessary component of this patient's treatment? A Ankle-foot orthotic B Insulin pump C Supplemental pancreatic enzymes D Multiple dental implants E Bone-conducting hearing aids

Answer E Bone-conducting hearing aids Explanation The patient has radiographic and clinical features of Treacher-Collins syndrome, or mandibulofacial dysostosis. It is caused by failure of neural crest cells to migrate into the 1st and 2nd branchial arch due to a gene mutation located on chromosome 5. As a result, craniofacial tissues such as cartilage, bone, and connective tissues fail to develop normally. Many infants have an associated cleft lip/palate. External ear abnormalities are associated with severe conductive hearing loss, requiring use of bone-conducting hearing aids for proper language development. The typical facial features are the result of malar hypoplasia and a cleft in the zygoma. The eyes have downward-slanting palpebral fissures. Colobomata along the lateral 3rdof the lower lid are associated with a striking absence of the eyelashes from the medial 2/3 of the lower eyelid.

A 2-day-old Amish female has detectable L-alloisoleucine, which was discovered during a routine newborn screening laboratory. Which of the following is the most appropriate dietary restriction for this patient? A Galactose, lactose B Phenylalanine C Fructose, sucrose, sorbitol D Phenylalanine, tyrosine E Branched-chain amino acids

Answer E Branched-chain amino acids Explanation This infant has detectable L-alloisoleucine, an abnormal amino acid that is diagnostic for maple syrup urine disease (MSUD), an autosomal recessive condition that has an increased incidence in certain Amish and Mennonite populations. Appropriate treatment is branched-chain amino acids restriction. Branched-chain amino acids include leucine, isoleucine, and valine. Fructose, sucrose, and sorbitol are restricted in hereditary fructose intolerance. Phenylalanine is restricted in phenylketonuria. Phenylalanine and tyrosine are restricted in diets of patients with tyrosinemia. Galactose and lactose are restricted in patients with galactosemia. Board Testing Point Know the dietary restrictions for patients with maple syrup urine disease (MSUD).

Soon after birth, a term male is noted to have noisy respirations that appear to be due, at least in part, to choanal stenosis, documented by difficulty passing a small feeding tube through either nares. He is also noted to have symmetrical midface hypoplasia caused by bony deficiency of the lateral portion of the orbital bones and bilateral zygoma and maxillary hypoplasia. Associated findings include a cleft palate, hypoplasia of the lower eyelids associated with absent lower eyelashes, and micrognathia. Which of the following complications is most likely to occur? A Bilious emesis B Conjugated hyperbilirubinemia C Obstructive uropathy D High-output heart failure E Conductive hearing loss

Answer E Conductive hearing loss Explanation The patient has clinical findings consistent with Treacher-Collins syndrome, or mandibulofacial dysostosis. Outer and middle ear malformations are common, causing a conductive hearing loss, the severity of which is associated with the extent of the associated malformations of the external ear and auditory ossicles. However, the inner ear is usually spared. Thus, patients benefit significantly from early assessment of hearing and prompt intervention with appropriately fitted hearing aids. Treacher-Collins syndrome is characterized by midface and mandibular dysmorphology, which gives rise to bilateral craniofacial defects, including bony hypoplasia, various degrees of choanal stenosis or choanal atresia, and cleft palate with or without cleft lip. The palpebral fissures are downward slanting, the lower eyelids hypoplastic, and the lower-eyelid eyelashes absent.

A 30-month-old developmentally delayed boy is noted to have recently experienced further neurologic deterioration in association with the development of communicating hydrocephalus. He is in the 5th percentile for height and has coarse facial features, a prominent forehead, and hypertelorism. Hepatosplenomegaly and generalized joint stiffness are also present. Which of the following complications is most likely to occur as this patient grows older? A Recurrent bowel obstruction B Renal tubular acidosis C Recurrent pulmonary emboli D Pancreatic insufficiency E Myocardial hypertrophy

Answer E Myocardial hypertrophy Explanation The patient has Hunter syndrome, an X-linked disorder also known as mucopolysaccharidosis (MPS) Type 2, one of several disorders caused by a deficiency of lysosomal enzymes responsible for the degradation of mucopolysaccharides or glycosaminoglycans (GAGs). All other types of mucopolysaccharidoses are inherited in an autosomal recessive manner. Insufficient levels of iduronate-2-sulfatase lead to gradual accumulation of degraded GAGs, thickening tissue, and progressively compromising cellular and organ function in the brain, bones, heart, lungs, liver, and spleen. More severely affected patients present by 2-4 years of age; clinical findings include short stature, multiple skeletal deformities, joint stiffness, thickened skin, hypertelorism, and coarse facial features. Communicating hydrocephalus often develops, leading to further neurological deterioration. GAG accumulation in the cardiac valve leaflets is common; valvular dysfunction, in combination with myocardial thickening, may lead to heart failure and/or coronary artery compromise. Obstructive airways disease and pulmonary hypertension are also common. Milder forms of the disease, in which intelligence is preserved, may not present until adolescence. A biosynthetic iduronate sulfatase (Elaprase) is approved for treatment in patients with Hunter syndrome to diminish organ size and improve joint mobility. The key in this question is to understand that the mucopolysaccaridoses result in an accumulation of GAGs in various tissues and organs. Thus, although recurrent bowel obstruction, pancreatic insufficiency, renal tubular acidosis, and recurrent pulmonary emboli can be seen in various metabolic conditions, only myocardial hypertrophy is directly a cause of the GAG accumulation and seen in those with Hunter syndrome.

A 1-day-old male infant is noted to have multiple congenital anomalies. On physical examination, there is microcephaly with narrow bifrontal diameter, cleft palate, wide-set eyes, ptosis, micrognathia, hypospadias, and 2/3-toe syndactyly. Mother had cell free DNA testing prenatally that was normal. Laboratory tests demonstrate low total cholesterol and elevated 7-dehydrocholesterol. The most likely etiology of this child's features is: A Menkes disease B Zellweger syndrome C Lesch-Nyhan syndrome D Tay-Sachs disease E Smith-Lemli-Opitz syndrome

Answer E Smith-Lemli-Opitz syndrome Explanation This infant has features of Smith-Lemli-Opitz syndrome (SLOS), an autosomal recessive disorder of cholesterol metabolism. SLOS is due to a deficiency of 7-dehydrocholesterol reductase causing an increase in plasma 7-dehydrocholesterol and a decrease in plasma cholesterol. Characteristic facial features include microcephaly with narrow bifrontal diameter, broad nasal tip with anteverted nares, hypertelorism and ptosis, cleft palate and micrognathia, and low-set ears. Other features include abnormal thumbs, postaxial polydactyly, overlapping fingers, 2/3-toe syndactyly, hypospadias, and ambiguous genitalia. Zellweger syndrome is an autosomal recessive disorder of peroxisomal biogenesis. Patients with this syndrome have hepatic and renal dysfunction. Characteristic features include large anterior fontanelle, high forehead, epicanthal folds, cataracts, broad nasal bridge, jaundice, and epiphyseal stippling. Diagnostic laboratory findings include elevated very long-chain fatty acids. Menkes disease is an X-linked disorder caused by impaired uptake of copper. After infancy, serum copper and ceruloplasmin levels are low. Characteristic facial features include full cheeks with sagging jowls and lips. Hair and eyebrows are sparse with little pigment and are easily broken. By 2-3 months of age, there is neurologic degeneration. This is a rare cause of subdural hematomas and retinal hemorrhages not associated with trauma. Tay-Sachs disease is an autosomal recessive disorder that has a higher prevalence among Ashkenazi Jews. It is caused by a deficiency of β-hexosaminidase and is diagnosed by demonstrating loss of enzymatic activity. There is a characteristic pattern of neurologic stagnation and regression beginning in the first year of life. Infants do not have dysmorphic features but do develop a characteristic cherry-red spot on the retina. Lesch-Nyhan syndrome is an X-linked disorder due to deficiency of the hypoxanthine guanine phosphoribosyltransferase (HGPRT) enzyme leading to elevated purines, including uric acid. Diagnosis can be made by demonstrating loss of enzymatic activity. There are no dysmorphic features. Characteristic neurologic findings include motor dysfunction that resembles cerebral palsy, cognitive and behavioral disturbances, and self-mutilation. Other findings include renal stones and gout.

An 18-year-old young man with a history of mild intellectual disability presents with the acute onset of shortness of breath associated with severe chest pain. An emergent evaluation reveals that his symptoms are the result of a pulmonary embolus. Which of the following physical findings is often identified in patients at increased risk of this and other thromboembolic complications? A Acoustic neuroma B Cavernous hemangioma C Limb deficiency D Axillary freckling E Subluxation of the ocular lens

Answer E Subluxation of the ocular lens Explanation Individuals with homocystinuria are at increased risk of thromboembolic events, including cerebrovascular occlusions, myocardial infarction, and pulmonary emboli. Homocystinuria also is associated with various clinical findings such as dislocation (subluxation) of the lenses in a downward direction. This disorder is commonly characterized by a marfanoid appearance, dry and lightly pigmented skin, coarse hair, pectus deformities, scoliosis, and generalized muscular hypotonia. Many patients are mildly intellectually disabled and exhibit aggressive behavior. Homocystinuria is inherited in an autosomal recessive manner. It is caused by a defect in the gene coding for cystathionine-β-synthase, which converts homocysteine into cystathionine during metabolism of dietary protein. This deficiency leads to an accumulation of homocysteine, some of which is reconverted into methionine. High levels of both methionine and homocysteine are detectable in the urine. Elevated levels of homocysteine cause damage to collagen and elastic fibers within connective tissue and marked platelet accumulation, increasing the risk of formation of vascular thrombi. About half of affected individuals improve with pyridoxine treatment, which enhances activity of the deficient enzyme. Close monitoring of dietary intake of protein is important in all patients, especially those not responding to pyridoxine therapy. The other answer choices are not seen in homocystinuria. Axillary freckling is a feature of neurofibromatosis Type 1. Cavernous hemangiomas can be seen sporadically, isolated, or as part of genetic conditions. Limb deficiency is seen in multiple genetic conditions. Acoustic neuroma can be a feature of neurofibromatosis Type 2.

Following an evaluation for chronic headache, an 18-year-old girl is diagnosed with neurofibromatosis Type 2. Which of the following is most likely to be associated with this disorder? A Intention tremor B Sixth nerve palsy C Loss of peripheral vision D Exophthalmos E Tinnitus

Answer E Tinnitus Explanation Tinnitus, unsteady gait, hearing loss, and/or facial weakness are common presenting signs and symptoms of bilateral acoustic neuromas (vestibular schwannomas). Such findings are characteristic of neurofibromatosis Type 2 (NF2). Bilateral acoustic schwannomas usually occur in late adolescence/early adulthood, with all having this feature by 30 years of age. Bilateral vestibular schwannomas are the most characteristic finding among patients with NF2, which makes up about 10% of all cases of neurofibromatosis. Diagnostic criteria for this disorder include a patient with bilateral acoustic neuromas, or unilateral acoustic neuroma with ≥ 2 of the following: meningioma, glioma, schwannoma, juvenile posterior subcapsular lens opacity (juvenile cataract), or cortical wedge cataract. Neurofibromas are usually not seen in patients with NF2. In contrast to neurofibromatosis Type 1 (NF1), malignant transformation of the nervous system lesions does not occur in NF2. Café-au-lait spots and axillary freckling, common in NF1, are often absent. Lisch nodules (pigmented hamartomas of the iris), common in NF1, do not occur in NF2. The gene for NF2 is located on the long arm of chromosome 22q1.11. The majority of cases result from a mutation and are inherited in an autosomal dominant pattern. Exophthalmos, intention tremor, sixth nerve palsy, and loss of vision are not seen in NF2, but have been reported in NF1. NF1 is a separate condition from NF2. Individuals with NF1 can present with multiple café-au-lait spots, axillary and/or inguinal freckling, Lisch nodules, optic gliomas, neurofibromas and/or plexiform neurofibromas, and osseous lesions. Mutations in the NF1 gene at 17q11.2 result in NF1.

The mother of a 4-day-old male infant reports the baby has had 4 episodes of nonbilious emesis today. The last 2 episodes were projectile in character. The pregnancy and delivery were normal for this term infant. He initially did well and was discharged on day 2 of life. On physical examination, he is sleepy. His respiratory rate is 60, and his pulse rate is 160. His anterior fontanelle is slightly sunken, and his mucous membranes are dry. The remainder of his exam is normal. The CBC is normal. His serum ammonia level is massively elevated, but serum pH and bicarbonate levels are normal. What is the most likely cause of this infant's signs and symptoms? A Methlymalonic acidemia B Pyloric stenosis C Sepsis D Maple syrup urine disease E Urea cycle defect

Answer E Urea cycle defect Explanation Neonates with inborn errors of metabolism are typically normal at birth. Subsequently, the most common presentation is metabolic acidosis, vomiting, lethargy, and poor feeding. Many of these disorders are lethal if appropriate care is not initiated immediately. The most common cause of significant hyperammonemia in infants is a defect in the urea cycle. Infants with urea cycle defects are not acidotic on presentation. Early in the course of the metabolic crisis, the ABG will be normal. Without treatment, as the ammonia rises, the patient will progress to respiratory alkalosis. Although organic acidemias, such as methylmalonic acidemia and propionic acidemia, may have hyperammonemia as well, metabolic acidosis will also be present, and ammonia levels are not as high. The key for this patient is that the ammonia is high and there is a normal pH and bicarb (i.e., lack of metabolic acidosis). This takes the organic acidemias off the differential diagnosis. If the patient remains untreated, he will develop respiratory alkalosis. Sepsis can present with vomiting, but the infant would have hyperthermia (or hypo), the CBC would be abnormal, and there would not be hyperammonemia. Vomiting may be severe and projectile, sometimes arousing suspicion for pyloric stenosis. Pyloric stenosis would not cause hyperammonemia. Maple syrup urine disease also presents with acute neonatal encephalopathy and vomiting, but it is not associated with hyperammonemia.

A 7-month-old boy presents to the emergency department as lethargic and unarousable. Findings on physical examination are normal except for enlarged liver. Laboratory tests reveal a serum glucose concentration of 10.0 mg/dL. The mother states that the child slept through the night for the first time last night. Family history is negative for any serious or chronic illnesses. You are considering an inborn error of metabolism. Which of the following is the most important lab test to order next? A Serum calcium B Serum sodium C Serum lactate D Serum ammonia E Urine ketones

Answer E Urine ketones Explanation This is a classic presentation for a glycogen storage disease (GSD), especially given the enlarged liver after sleeping through the night for the first time. However, some of the fatty acid oxidation defects may present primarily with hypoglycemia and minor liver enlargement, especially if there is a concomitant minor viral illness. The urine ketones will help with initial differentiation. No or few ketones would be most consistent with a fatty acid oxidation defect, but excess ketones are most consistent with a glycogen storage disease. Serum ammonia may be elevated in fatty acid oxidation defects but is not always, so ammonia is not as useful as ketones. Serum lactate will be elevated regardless of the underlying cause, because the baby is hypoglycemic and using anaerobic metabolism for energy. When glucose is not available, the body uses protein (via alanine) to regenerate pyruvate, which in turn generates lactate. Fatty acids generate ketones, which can be used to generate lactate. Serum calcium and serum sodium are typically normal and not helpful as diagnostic lab tests. Board Testing Point Recognize the clinical presentation for glycogen storage disease and know the appropriate diagnostic tests.

A 4-month-old male is seen in the emergency department for recurrent epistaxis. Review of the medical record shows poor weight gain over the past 2 months. On physical examination, the abdomen is protuberant with the liver edge palpated at the iliac crest. There is no splenomegaly. There is radiographic evidence of rickets. Laboratory studies reveal normal blood glucose, mild anion gap metabolic acidosis, hypokalemia, prolonged PT/aPTT, and mild elevations of liver transaminases. Urine is negative for ketones and reducing substances. Which of the following is the best next test? A Plasma lipid profile B Liver biopsy C Lysosomal enzyme analysis D Plasma acylcarnitine profile E Urine succinylacetone level

Answer E Urine succinylacetone level Explanation This patient has features of tyrosinemia Type I (hepatorenal tyrosinemia). This form of tyrosinemia is autosomal recessive and is characterized by hepatic and renal dysfunction, without hypoglycemia. Failure to thrive, hepatomegaly with hepatoblastoma, and liver failure are the most common presentations. There is renal tubular acidosis resembling Fanconi syndrome as well as x-ray findings of rickets. This disorder is treated with 2-(2-nitro-4-trifluoro-methylbenzoyl)-1,3-cyclohexanedione (NTBC). The diagnostic compound for tyrosinemia Type I (succinylacetone) is concentrated in the urine. Succinylacetone is a toxic compound and leads directly to the most serious complications of the disorder. The level of succinylacetone correlates with the severity and level of control of this disorder. NTBC administration prevents production of this toxic compound. Fatty acid oxidation disorders (FAODs) are diagnosed by acylcarnitine profile. Absence of both urinary-reducing substances and ketones in an infant with hypoglycemia suggests an FAOD. Medium-chain acyl-CoA dehydrogenase (MCAD) deficiency is the most common FAOD and is associated with elevated medium-chain acylcarnitines (C8, C8:1, and C10:1). The characteristic presentation is fasting-induced lethargy and hypoglycemia. A plasma lipid profile is useful in the diagnosis of genetic dyslipidemias. In addition, hyperlipidemia is associated with glycogen storage disease (GSD). The prototype is Type I GSD. The characteristic time of presentation is at 3-4 months of age when parents are attempting to have the infant sleep through the night (increased length of fasting). Clinically, there is hepatomegaly, failure to thrive, and hypoglycemic seizures. Hypoglycemia is associated with urine ketones but absent reducing substances. There is no response to glucagon. In addition to hypoglycemia, there is lactic acidosis, hyperuricemia, and hyperlipidemia. Liver transaminases are typically normal. Because one of the major complications of liver biopsy is hemorrhage, it would be a mistake to pursue this in a coagulopathic patient. Lysosomal enzyme analysis is used for lysosomal storage disorders such as mucopolysaccharidoses (MPS). The prototypes, MPS Type I (Hurler syndrome) and MPS Type II (Hunter syndrome), have hepatomegaly but do not have renal tubular acidosis or rickets.

How do patients with biotinidase deficiency present and how do you screen them?

Biotinidase levels screen for biotinidase deficiency. This disorder presents in young children with the slow evolution of neurologic abnormalities, including seizures, hypotonia, ataxia, developmental delays, vision problems, hearing loss, alopecia, and a skin rash.

How do patients with Menkes disease present and how do you screen them?

Ceruloplasmin levels, in conjunction with copper levels, screen for Menkes disease. This disorder presents with a period of normal development in early infancy, followed by developmental regression, coarse, kinky hair (pili torti), and tortuosity of the carotid arteries and vasculature of the brain.

Of the following, the isolated clinical finding that MOST warrants referral to a genetics specialist in a neonate without any obvious facial dysmorphology is? A. clubfoot B. deafness C. horseshoe kidney D. polydactyly of the hand E. single transverse palmar crease

Deafness. Up to 60% of cases of congenital and early-onset hearing loss are caused by genetic factors. In more than 400 genetic syndromes, hearing loss is a feature and more than 100 genes are associated with nonsyndromic genetic hearing loss. Of the patients with hearing loss from a genetic cause, 70% are nonsyndromic and can have an autosomal dominant (15%), autosomal recessive (80%), X-linked (1%), or mitochondrial (1%) inheritance pattern. It is important to take a detailed 3-generation family history to look for a particular genetic etiology. Approximately 30% of genetic hearing loss is syndromic; thus, a thorough physical examination is essential in evaluating a child with hearing loss. Practitioners should be alert to any of the following abnormal findings: Abnormal pigmentary anomalies Asymmetry of facial features Cardiac anomalies (especially long QT syndrome) Cleft lip or palate Heterochromia of the irides Malformation of an auricle Microcephaly Renal abnormalities Skin tags/ear pits near the auricle Thyroid disease Some physicians advocate for temporal bone imaging in these cases to look for an enlarged vestibular aqueduct, which would be suggestive of Pendred syndrome. If a particular syndrome is suspected, gene sequencing specific to that syndrome is warranted. If the patient is nonsyndromic, then it is commonly recommended to begin with GJB2 and GJB6 gene mutation analysis. Fifty percent of autosomal recessive nonsyndromic hearing loss is due to GJB2 and GJB6 gene mutations, which encode for connexin 26 and connexin 30 gap junction proteins. If the result of this testing is negative, it is appropriate to perform a hereditary hearing loss next-generation sequencing panel that can test for many known nonsyndromic gene mutations in a single blood test. Acquired hearing loss can be caused by various infectious diseases. Cytomegalovirus (CMV) is the most common nongenetic cause of hearing loss in children; therefore, a urine CMV test should be performed at the same time as genetic testing in infants presenting with congenital hearing loss. The CMV testing is most accurate when performed in the first 6 weeks after birth. Meningitis is another leading cause of acquired permanent hearing loss, whereas otitis media can result in reversible hearing loss. It is also important to inquire about any history of ototoxic drug use (eg, aminoglycosides, cisplatin), birth hypoxia, or hyperbilirubinemia. The early identification of hearing loss is important because early intervention has clearly been shown to be effective in promoting speech and language acquisition in deaf and hard-of-hearing children. As a result, newborn hearing screening is now mandated across the United States. However, not all types of hearing loss are present at birth. Hearing loss can develop throughout childhood and adolescence, making hearing screening a vitally important aspect of health supervision. Clubfoot, horseshoe kidney, or polydactyly of the hand presenting as an isolated finding would not necessitate a genetics referral, because these anomalies are commonly multifactorial and on their own do not suggest a genetic etiology. The isolated finding of a single transverse palmar crease can be seen in normal individuals, but in pediatrics, trisomy 21 must be considered, especially if other associated dysmorphology is present. Children with intellectual disability, autism spectrum disorder, or multiple congenital anomalies warrant consideration of a genetic diagnosis.

What's the defect in Type 1 glycogen storage disease (von Gierke disease)?

Defect in glucose-6-phosphatase enzyme

A 14-year-old boy with a history of ADHD, emotional lability, and a learning disorder is found to have bilateral scarring of tympanic membranes, macroorchidism, joint hypermobility, and pes planus during physical examination. Which of the following best describes the most likely associated facial features of this patient? A Wide-set eyes, upturned nose, large maxilla, prominent philtrum, and pointed chin B Flattened mid-face, low-set ears, shortened nose, thin upper lip, indistinct philtrum, and pointed chin C Widely-spaced eyes, low-set ears, webbing of the neck, shieldlike chest, pectus deformity, and increased carrying angle of the arms D Upward slanting palpebral fissures, epicanthal folds, flat nasal bridge, and small, whitish spots on the periphery of each iris E Prominent forehead; long, narrow face; prominent jaw; and protuberant ears

E Prominent forehead; long, narrow face; prominent jaw; and protuberant ears Explanation The patient has clinical findings consistent with fragile X syndrome—the most common form of familial intellectual disability in males. Fragile X syndrome is caused by expansion of a DNA segment consisting of a single trinucleotide gene sequence, cytosine-guanine-guanine (CGG), on the X chromosome. The normal number of CGG repeats is 5-40. Patients with the full mutation have > 200 CGG repeats in the FMR1 gene, which codes for a protein required for normal development of neural synapses. The classic facial appearance is described as a prominent forehead; long, narrow face; prominent jaw; and protuberant ears. It is also associated with a high arched palate, dental crowding, and malocclusion. Strabismus and chronic otitis media are common. Affected patients often have autistic-like mannerisms and are anxious, socially awkward, and have frequent temper outbursts associated with emotional lability. Many also have learning disabilities, language processing disorders, and attention deficit-hyperactivity disorder. Seizures occur in up to 20% of affected patients. Macroorchidism is evident by adolescence, with mean testicular volume usually measuring about 50 mL, or twice the normal testicular volume. Complications associated with connective tissue abnormalities, such as progressive dysfunction of hyperextensible joints and development of mitral valve prolapse, are common.

The parents of a 9-year-old girl are contacted after their daughter evidently tried to get into a car driven by the mother of another child at her school because she heard that they were planning to get an ice cream cone on their way home. Reportedly, she did not know the other child or her mother. Her parents are distraught, stating that their daughter is "overly friendly," and has no fear of strangers. On several occasions, she had placed herself at potential risk because she is so outgoing, often initiating contact with complete strangers. Which of the following is most often associated with these behavioral characteristics and traits? A Turner syndrome B Rett syndrome C Autism spectrum disorder D Angelman syndrome E Williams syndrome

E Williams syndrome Explanation Williams syndrome is a sporadic disorder associated with a microdeletion on chromosome 7q11.23, in which patients are often very friendly, possessing a "cocktail party" personality, and frequently described as "never having met a stranger." However, although affected individuals are engaging and very likable, they may lack insight into the potential dangers associated with interacting with total strangers in an outgoing and affectionate manner. Dysmorphic features in Williams syndrome include an elfin-like facies, often associated with strabismus, small peg-like teeth, wide-set eyes, upturned nose, large philtrum, and pointed chin. Learning disabilities, often associated with below-average IQ, are common, especially in areas that require skills in visual-spatial perception and integration. Supravalvular aortic stenosis, characterized by a loud systolic ejection-type murmur heard best at the right 1st intercostal space, is often present, as is peripheral pulmonic stenosis. Some patients with Williams syndrome also have idiopathic hypercalcemia of infancy and maintain high plasma calcium concentrations leading to hypercalciuria, delayed clearance of calcium from the plasma, and hypercalcemia-induced nephrocalcinosis associated with renovascular hypertension.

What's the definitive diagnosis for Type 1 glycogen storage disease (von Gierke disease)?

Gene based mutational analysis

conditions that can present with lissencephaly.

Individuals with Miller-Dieker syndrome have lissencephaly as well as the microcephaly, delays, seizures, and mild facial malformations.

What is MCAD?

Medium-chain acyl coenzyme A dehydrogenase (MCAD) deficiency is an autosomal recessive fatty acid oxidation disorder that presents with elevations in C6, C8, and C10 acylcarnitines on newborn screening tests. Children with MCAD are normal at birth, typically presenting between 3 and 24 months of age with an episode of hypoketotic hypoglycemia, vomiting, and lethargy triggered by a minor illness. Children with MCAD are normal at birth, typically presenting between 3 and 24 months of age with an episode of hypoketotic hypoglycemia, vomiting, and lethargy triggered by a minor illness.

How do patients with Mucopolysaccharidoses present and how do you screen them?

Mucopolysaccharidoses are identified through lysosomal enzyme screening and urine glycosaminoglycans. These disorders typically present with a slowly progressive coarsening of facial features, joint stiffness, and developmental regression.

Conditions associated with radial aplasia

Patients with bilateral radial aplasia often have associated congenital heart disease (in 30-35 % of cases) and experience episodes of severe hemorrhage secondary to thrombocytopenia—known as thrombocytopenia-absent radius (TAR) syndrome.

During an evaluation following a 10-minute generalized tonic-clonic seizure, a 7-year-old boy, with no previous history of a seizure disorder, is noted to have 9 café-au-lait macules over his anterior and posterior trunk. All lesions measure between 7 and 12 mm. Which of the following bony abnormalities is most likely to be identified upon continued evaluation? A Ganglion cyst B Osteochondroma C Osteoid osteoma D Chondroblastoma E Sphenoid dysplasia

Sphenoid bone dysplasia in neurofibromatosis type 1 is characterized by progressive exophthalmos and facial disfiguration secondary to herniation of meningeal and cerebral structures.

A 2-year-old boy visits your office for follow-up care after his recent ophthalmologic evaluation for retinal detachment and upward lens subluxation. He is tall for his age and has an increased arm span-height ratio. His upper segment-lower segment ratio is normal for his age. Otherwise, his medical history is unremarkable. His father is 6 ft 2 in and has an increased arm span-height ratio, reduced upper segment-lower segment ratio, high-grade myopia, lens subluxation, flat feet, scoliosis, and a positive wrist and thumb sign. The boy's paternal uncle has aortic root dilation and reduced elbow extension with a negative wrist and thumb sign. His paternal grandmother has aortic root dilation and myopia. Consanguinity is denied. Of the following, the genetic term that BEST describes the familial presentation in this case is A. autosomal recessive inheritance B. inbreeding C. reduced penetrance D. sex-limited expression E. variable expression

The family in this vignette has an autosomal dominant disorder with variable expression. Expressivity is a term used to describe the phenotypic expression of a disease among individuals with a particular genotype. The family members in this vignette have Marfan syndrome, which is associated with pathogenic mutations in FBN1. Variable expression occurs when the severity and presentation of disease manifestations differs among individuals with the same genotype. The boy in this vignette has retinal detachment and lens subluxation and his paternal grandmother has aortic root dilation and myopia, demonstrating variable systemic involvement among members of the same family. Another common term in genetics is incomplete penetrance. Penetrance is the probability that a gene will manifest clinically in an individual with a particular genotype. Incomplete or reduced penetrance is an all-or-none phenomenon. Incomplete penetrance occurs when an individual with a specific genotype does not express any signs or symptoms of the disorder. Phenotypic expression can be impacted by modifier genes, aging, or environmental factors. Penetrance is an all-or-none phenomenon. It describes the percentage of people with a predisposing phenotype who actually have clinical effects, even to a small degree, vs the individuals with the same phenotype with no clinical effects. An example is reduced penetrance in individuals with split-hand deformity, a type of ectrodactyly. Some individuals will have the split hand and some will not despite possessing the same genotype. Only 70% of individuals with this gene mutation exhibit the clinical defect. The patterns demonstrated in single-gene disorders in pedigrees generally depend on several factors. A disorder is considered autosomal dominant when only one chromosome of the pair carries the abnormal allele and the other chromosome carries a normal allele, but the single abnormal allele is enough to yield an abnormal phenotype in the affected individual. A disorder is considered autosomal recessive when both chromosomes of the pair carry the abnormal alleles, as in cystic fibrosis or sickle cell disease. A gene mutation on each chromosome in the pair is necessary for the individual to have the disorder. This type of inheritance is called autosomal recessive inheritance. If an individual only carries one gene mutation on a single chromosome that is associated with an autosomal recessive disorder, the individual will be a carrier for the disorder but unaffected. The chromosome location indicates if the gene is located on an autosome (chromosomes 1 to 22) or on a sex chromosome (chromosomes X or Y). Autosomal disorders generally affect male and female individuals equally, with the exception of sex-limited disorders. In autosomal dominant disorders, an affected individual has a 50% chance of passing the disorder to his or her offspring. Some disorders have sex-limited expression despite the fact that the disorder is on an autosome rather than a sex chromosome; the disorder will only express itself in a particular sex. Examples of sex-limited expression are male-pattern baldness and male-limited precocious puberty. Heterozygous females do not manifest the disorder in sex-limited expression disorders. Inbreeding is a term used to describe mating between individuals from the same small population. Inbreeding could be caused by geographical or cultural reasons. The parents may not consider themselves related but could have a common ancestor within the previous several generations. This shared ancestry predisposes individuals to increased carrier rates for recessive disorders.

You are seeing a 5-day-old newborn in your office for an initial health supervision visit. The mother reports an unremarkable pregnancy and delivery, with a brief period of hypoglycemia after delivery that resolved quickly with the first feeding. The newborn has been feeding well without signs of lethargy since that time. His physical examination is unremarkable. The mother is anxious, having had an infant 10 years ago who died of sudden infant death syndrome after a 2-day respiratory illness while the family was living abroad. You received notification today that this newborn's screening test was flagged for elevations of C6, C8, and C10 acylcarnitines, with the C8 level most significantly elevated. As prescribed in the ACT algorithm published by the American College of Medical Genetics, you order a blood glucose and electrolyte levels, blood gas, liver function tests, urine organic acids, urine acylglycines, and a plasma acylcarnitine profile. The laboratory results reveal: Plasma acylcarnitine profile: elevated C8 Urine organic acids: slightly elevated dicarboxylic acids Urine acylglycines: elevated hexanoylglycine Blood glucose, liver function tests, blood gas, and electrolyte levels were all normal. Of the following, this newborn's MOST likely diagnosis is A. carnitine-palmitoyltransferase-2 deficiency B. maple syrup urine disease Correct C. medium-chain acyl-coenzyme A dehydrogenase deficiency D. phenylketonuria E. propionic aciduria

The laboratory abnormalities for the newborn in the vignette are consistent with medium-chain-acyl-coenzyme A dehydrogenase (MCAD) deficiency, an autosomal recessive fatty acid oxidation disorder. Elevations in C6, C8, and C10 acylcarnitines are the diagnostic laboratory findings. Medium-chain-acyl-coenzyme A dehydrogenase is the most common fatty acid oxidation disorder. Children with this disorder are normal at birth and typically present between 3 and 24 months of age, with an episode of hypoketotic hypoglycemia, vomiting, and lethargy triggered by a minor illness. Seizures, hepatomegaly, and liver dysfunction are common and, if not treated appropriately, can lead to coma and death. Prognosis can be excellent if appropriate management is implemented. Management involves the prevention of hypoketotic hypoglycemia events. Children are provided with frequent feedings to avoid any prolonged period of fasting, especially with intercurrent illness. Young children are placed on a low-fat diet (< 30% fat) and should ingest 2 g/kg of uncooked cornstarch at bedtime to ensure adequate glucose during the overnight period. During periods of illness, affected children should be treated with simple carbohydrates by mouth or intravenously, if necessary, to maintain an anabolic state. A family history of sudden death in childhood, especially in the first 2 years of life, or Reye syndrome in siblings is a clue suggesting MCAD. The inclusion of MCAD on newborn screening is critical in preventing morbidity and mortality in affected children, allowing the implementation of recommended management guidelines as early as the newborn period, thereby increasing the likelihood of a normal lifespan and outcome. In the 1960s, Dr. Robert Guthrie developed newborn screening to identify newborns with phenylketonuria (PKU). Phenylketonuria is an inherited metabolic disorder that can result in severe intellectual disability, unless appropriate dietary interventions are initiated in early infancy. A primary goal of newborn screening is to detect newborns with serious but treatable disorders. Early identification allows timely implementation of interventions necessary to prevent or ameliorate negative consequences of these disorders, leading to significant reductions in death and disease-associated disabilities. The specific disorders tested on newborn screening varies by state, but all states have expanded the number of diseases included, and now routinely test for 30 or more metabolic, endocrine, and genetic disorders, including PKU, galactosemia, organic acidemias, fatty acid oxidation disorders, congenital adrenal hyperplasia, congenital hypothyroidism, and sickle cell disease. The American College of Medical Genetics has developed Newborn Screening ACT (ACTion) sheets for many genetic and metabolic disorders identified on newborn screening. These sheets guide physicians on how to communicate with the family and determine the appropriate referrals and laboratory testing needed when an infant screens positive for a disorder. This allows healthcare personnel to act in a timely manner to confirm a suspected diagnosis and initiate treatment as soon as possible, improving outcomes. Most primary care physicians will work in collaboration with their state newborn screening program. The state program will manage all cases of newborns with abnormal screening by contacting their families and physicians and coordinating management in an expedient manner. If an inherited disorder is identified, the family may be referred for genetic counseling about recurrence risk.

You are called to the newborn nursery to evaluate a 3-day-old female newborn with worsening neurologic status. The full-term newborn was delivered via repeat cesarean delivery after an unremarkable pregnancy. She was initially doing well after birth. At approximately 12 to 24 hours of age, the mother noted a sweet, caramel-like odor. At 2 days of age, the newborn was feeding poorly, becoming irritable, and then developed drowsiness that progressed to lethargy, intermittent apnea, opisthotonus, and hypertonia. On day 3 after birth, the newborn developed "bicycling" movements of the legs. On physical examination, the newborn appears mildly dehydrated, lethargic, and hypertonic. Initial laboratory workup shows mild metabolic acidosis (pH = 7.3) and a blood ammonia level of 280 μg/dL (200 μmol/L). A complete blood cell count, lactate, calcium, and glucose are normal. A sepsis workup is initiated with cultures pending. You suspect an inborn error of metabolism. Of the following, the BEST next test to confirm your suspected diagnosis is A. biotinidase level B. ceruloplasmin level C. lysosomal enzyme screen Correct D. serum amino acids E. very-long-chain fatty acids

The newborn in the vignette has maple syrup urine disease (MSUD). By 12 to 24 hours of age, neonates with classic MSUD will have a maple syrup odor that is evident, especially in the cerumen. Plasma concentrations of branched-chain amino acids (isoleucine, leucine, and valine) and alloleucine will be elevated on serum amino acid analysis. Branched-chain hydroxyacids and ketoacids are evident on urine organic acid analysis. By 2 to 3 days of age, affected infants experience ketonuria, fussiness, and poor feeding. By 4 to 7 days of age, encephalopathy ensues, with opisthotonus, intermittent apnea, and lethargy progressing to respiratory failure and coma. Maple syrup urine disease is an inborn error of metabolism belonging to the subtype known as organic acidemias. Organic acidemias are characterized by the excretion of non-amino organic acids in the urine, caused by an enzymatic deficiency in specific steps involved in amino acid catabolism. Other organic acidemias include: Glutaric acidemia type I Isovaleric acidemia, homocystinuria Methylmalonic acidemia Propionic acidemia 3-hydroxy-3-methylglutaryl-Coenzyme A (HMG-CoA) lyase deficiency All are inherited in an autosomal recessive manner. Typically, newborns with these disorders appear well during the first few days after birth, with rapid decompensation to an encephalopathic state if not quickly identified. Common laboratory abnormalities include: Acidosis Elevated liver function tests Hyperammonemia Ketosis Low blood glucose Neutropenia Recommended laboratory tests in the setting of a suspected organic acidemia include serum amino acids, urine organic acids, ammonia level, and a plasma acylcarnitine profile. Patients have improved outcomes if the disorder is identified in the first 10 days after birth and appropriate treatment and dietary restrictions are implemented. With the advent of newborn screening programs using tandem mass spectrometry, MSUD is commonly identified shortly after birth, thus allowing for life-saving early treatment. Management includes dietary leucine restriction, specially manufactured branched chain amino acid-free foods, supplementation with isoleucine and valine, and intermittent biochemical monitoring. Care also includes clinical evaluations by a team specializing in metabolic disorders, including a biochemical geneticist, genetic counselor, and a metabolic dietitian. These patients are at increased risk for metabolic decompensation during periods of catabolic stress, such as intercurrent illness, and may require frequent hospitalization to manage the metabolic disorder appropriately and prevent the serious complication of brain edema.

A 17-year-old male is found to have bilateral cataracts after he presented complaining of changes in vision. On further questioning, he also complains of bilateral tinnitus and progressive hearing loss. Cranial MRI identifies enhancing lesions in the region of both internal auditory canals, which extend into the cerebellopontine angle Which of the following is the most likely diagnosis in this patient? A Tuberous sclerosis B Rhabdomyosarcoma C Langerhans cell histiocytosis D Acquired toxoplasmosis E Neurofibromatosis Type 2

The patient has clinical findings, confirmed by MRI, of bilateral acoustic neuromas, benign schwannomas of the vestibular portion of the 8th cranial nerve consistent with the diagnosis of neurofibromatosis Type 2 (NF2). NF2 is an autosomal dominant disorder associated with multiple tumors of the nervous system; it accounts for about 10% of all cases of neurofibromatosis. The NF2 gene, responsible for the production of merlin (a tumor suppressor), is located near the center of the long arm of chromosome 22q1.11. The clinical diagnosis of NF2 is based upon the presence of: Bilateral vestibular schwannomas Unilateral vestibular schwannoma accompanied by 2 of the following: meningioma, schwannoma, neurofibroma, glioma, cataract in the form of subcapsular lenticular opacities, or cortical wedge cataract Associated signs and symptoms include hearing loss, visual disturbances, tinnitus, cranial nerve abnormalities, gait disturbances, and/or headache. Café-au-lait spots and/or cutaneous neurofibromas are less common in patients with NF2 than in those with NF1. Tuberous sclerosis presents with brain tumors, of which there are 3 types: cortical tubers, which form on the surface of the brain but may also appear in the deep areas of the brain; subependymal nodules (SEN), which form in the walls of the ventricles; and subependymal giant cell astrocytomas (SEGA), which develop from SEN and grow such that they can block the flow of fluid within the brain, cause a buildup of fluid and pressure, and lead to headaches and blurred vision. This disorder usually presents in the 1st year of life, many times with difficult-to-treat infantile spasms. Acquired toxoplasmosis presents with either single (30%) or multiple nodular lesions. More commonly, the lesions are cavitating lesions with ring enhancement and surrounding edema. Toxoplasmosis has a propensity to involve the basal ganglia in 75%; in other cases, the nodules are scattered throughout the brain at the gray matter-white matter junction. Langerhans cell histiocytosis (LCH) is a condition due to abnormal proliferation of histiocytes. Individuals can have unifocal or multifocal disease. LCH causes inflammation which results in fever, swelling, pain, lethargy, and weight loss. The most frequent area of disease is in the bone. The skull is most frequently affected, followed by the long bones of the upper extremities and flat bones. Much less involved is the CNS. Rhabdomyosarcoma (RMS) is an aggressive and malignant form of cancer that arises from the skeletal muscle. RMS can occur in any site on the body but is primarily found in the head (parameningeal and other locations), neck, orbit, genitourinary tract, genitals, and extremities.

A 26-month-old boy is admitted for further evaluation after initial presentation in the emergency department with blood pressure of 170/110 mmHg and complaints of headache, vomiting, and irritability. After extensive evaluation, renal arteriogram shows evidence of severe unilateral renal artery stenosis. Findings on physical examination include a loud systolic ejection murmur heard best over the 1st right intercostal space without associated clicks or a diastolic murmur. Which of the following is most likely to be identified during additional evaluation of this patient? A Nasal polyps and meconium ileus B Unilateral facial nevus involving the forehead and eyelid C Anotia (absence of the pinna and ear canals) D Heterochromia and hearing loss E Elfin-like facies

The patient has hypertension due to renal artery stenosis and findings on cardiac auscultation consistent with supravalvular aortic stenosis. These findings are often observed in patients with Williams syndrome, characterized by elfin-like facial features, which include a broad forehead; medial eyebrow flare; a shortened, upturned nose associated with a flattened nasal bridge; an elongated philtrum; and a wide mouth with prominent, full lips. Many individuals also have both hypertelorism and strabismus. Peripheral pulmonary artery stenosis is also common. Williams syndrome is due to a deletion in region 7q11.23 on chromosome 7, which includes the elastin gene. Idiopathic hypercalcemia is common in infancy and during early childhood. Hypercalciuria persists for a longer period of time and often leads to nephrocalcinosis. Multivitamin preparations should be avoided because of the possible deleterious effects of vitamin D. Cognitive, motor, and language delays are universal. The majority of children are ultimately diagnosed with intellectual disability. Most demonstrate a relative strength in language skills and auditory memory with weakness in visuospatial cognition. Many are diagnosed with attention deficit disorder. Although acquisition of language skills is delayed, older children and adolescents often talk excessively. Anxiety and hypersensitivity to sound lead to additional behavioral problems in the school setting. Individuals are frequently described as overly friendly, empathetic, and outgoing with strangers (sometimes placing themselves at risk of potential harm). Heterochromia and hearing loss are seen in individuals with Waardenburg syndrome. Moderate to profound hearing loss can occur in one or both ears and is usually congenital. These individuals also have patches of depigmentation on their skin and a white forelock or premature graying of the hair. Anotia can be isolated but can also be seen in Treacher Collins or Goldenhar syndromes. Treacher Collins is characterized by downward-slanting eyes, micrognathia with or without cleft palate, conductive hearing loss, underdeveloped zygoma, drooping part of the lateral lower eyelids with eyelid coloboma, and malformed or absent ears. Goldenhar is a rare congenital condition characterized by incomplete development of the ears, nose, soft palate, lip, and mandible due to anomalous development of the first and second branchial arch. Limbal dermoids and periauricular skin tags are also common. Nasal polyps and meconium ileus are seen in cystic fibrosis. Cystic fibrosis is characterized by the buildup of thick, sticky mucus that can damage many of the body's organs. The disorder's most common signs and symptoms include progressive damage to the respiratory system with frequent infections, inflammation, and fibrosis. Frequent sinus infections and mucus buildup result in nasal polyps. Digestive problems are also common. Meconium ileus is often a presenting sign in babies. Blockage of the pancreatic duct prevents digestive enzymes and insulin from being released as well as causing other issues, including malnutrition and poor growth. Unilateral facial nevus involving the forehead and eyelid. This finding is seen in Sturge-Weber syndrome. This condition is a neurocutaneous condition with port wine stain of the ophthalmic branch of the trigeminal nerve of the face, glaucoma, seizures, and ipsilateral leptomeningeal angiomas.

A 14-year-old adolescent boy with moderate intellectual disability and autism presents to your office for a health supervision visit. On physical examination, you note macrocephaly, long face, prognathism, and large testes. His family history is significant for a younger sister with mild intellectual disability and a maternal male first cousin with moderate intellectual disability. No genetic testing has been performed. The patient's mother, who is cognitively normal, experienced premature ovarian failure. The family pedigree is shown in Item Q176 . Of the following, the BEST next test to confirm the diagnosis would be Correct A. FMR1 molecular analysis B. genomic microarray C. MECP2 molecular analysis D. plasma phenylalanine concentration E. Prader-Willi methylation testing

The patient in the vignette has fragile X syndrome, one of the most common forms of inherited intellectual disability. Fragile X syndrome results from an abnormality in a gene on the X chromosome. More than 99% of affected individuals have a loss-of-function mutation in FMR1 because of an increased number of CGG trinucleotide repeats (> 200) and abnormal methylation of the FMR1 gene. Diagnosis is confirmed on FMR1 molecular analysis. Fragile X syndrome is an X-linked autosomal-dominant, trinucleotide repeat disorder characterized by a phenomenon in genetics known as anticipation. With anticipation, a condition tends to become more severe and manifests at an earlier age as it is passed down from one generation to the next. This is due to expansion of an unstable trinucleotide repeat that is prone to errors during cell division. In fragile X syndrome, anticipation occurs when an intellectually normal premutation carrier, an individual with 55 to 200 repeats, transmits an unstable FMR1 allele to their child. This unstable allele allows the parent's premutation to expand into a full mutation, with more than 200 CGG repeats, thus yielding an affected child. The premutation usually expands through the mother; it tends to be stable if passed down by the father. Other disorders with anticipation include myotonic dystrophy and Huntington disease. Boys with a full fragile X mutation (> 200 CGG repeats) will have moderate-to-severe intellectual disability, whereas affected girls will have only mild intellectual disability or learning disabilities and can be intellectually normal in 50% of cases. Twenty-five percent of patients will also have autism spectrum disorder. Affected boys have a characteristic dysmorphology that includes a long face, large protruding ears, prominent forehead, prognathism, macrocephaly, and postpubertal macro-orchidism. In full-mutation individuals, the dysmorphology becomes more evident as the child ages, whereas premutation carriers have normal intellect and appearance. Male premutation (55-200 CGG repeats) carriers have an increased incidence of fragile X-associated tremor/ataxia syndrome (FXTAS), which resembles a Parkinson-like disorder with intention tremor, gait ataxia, and eventually dementia. Female premutation carriers are at risk for primary ovarian insufficiency, with cessation of menses before age 40 years. The pedigree in the vignette is consistent with an X-linked dominant disorder, with both males and females affected, but the males are affected to a much greater degree. The American Academy of Pediatrics and the American College of Medical Genetics recommend chromosomal microarray and fragile X testing on all children with intellectual disability.

Trisomy 21 with a Robertsonian translocation

The risk for trisomy 21 with a Robertsonian translocation 14:21 inherited from an infant's mother is 10% to 15%, but only 1% to 2% if inherited from the father. The recurrence risk for an unbalanced translocation, if inherited, is unaffected by parental age. Most cases of trisomy 21 result from meiotic nondisjunction yielding 3 complete copies of chromosome 21, rather than a translocation. This risk increases with maternal age, most significantly after age 35 years Understand the risk factors of having another child with trisomy 21 when the mother is a balanced translocation carrier Understand the risk factors associated with subsequent pregnancies when an infant is born with a translocation chromosome abnormality

Paternally acquired chromosome 15 anomaly

Trader willi syndrome - PA

A 4 mo patient with poor growth and an enlarged abdomen for follow-up care after an emergency department visit. The parents state that they have taken the infant to the emergency department on several occasions for low blood sugar after the baby began to sleep through the night. One of these episodes was associated with a seizure. The infant has doll-like facies with fat cheeks, protuberant abdomen, hepatomegaly, renomegaly, short stature, and thin arms and legs. No splenomegaly is noted. You order laboratory tests that reveal elevated lactate, uric acid, and lipid levels, along with a low serum glucose level. Liver transaminases, complete blood cell count, and hemoglobin A1C are within the normal range. You order an echocardiogram that rules out a congenital heart defect with associated congestive heart failure. The family history is unremarkable, and the infant's developmental milestones are appropriate. What's the diagnosis?

Type 1 glycogen storage disease (von Gierke disease)

How do patients with peroxisomal disorders present and how do you screen them?

Very-long-chain fatty acids screen for peroxisomal disorders, which present with a slow progression of hypotonia, poor feeding, dysmorphic facies, seizures, hepatic dysfunction, retinal dystrophy, and sensorineural hearing loss.

Down Syndrome - evaluating a child with down syndrome

When first evaluating a newborn with suspected trisomy 21, the pediatrician should perform a comprehensive physical examination to assess for associated anomalies and a fluorescence in situ hybridization analysis for trisomy 21 to quickly confirm the diagnosis for the family. A high resolution chromosome analysis to assess the mechanism of the trisomy 21 is also required. This analysis will reveal if the trisomy 21 is caused by a complete extra chromosome 21 (sporadic trisomy 21) or by an unbalanced translocation; this information allows the family to be informed of recurrence risk. Fluorescence in situ hybridization analysis can indicate that an extra copy of chromosome 21 is present, but it cannot detect a translocation. Advancing maternal age increases the risk for sporadic trisomy 21. Physical examination of the newborn with trisomy 21 should include careful evaluation for cataracts by looking for a red reflex. Auscultation for a cardiac murmur and pulse oximetry are important initial evaluations for cardiac disease. The child should be observed for stridor, wheezing, or noisy breathing that could indicate cardiorespiratory anomalies or intestinal atresias. A careful history for feeding problems, gastroesophageal reflux, constipation, apnea, bradycardia, cyanosis, or other respiratory difficulties is also needed. A brainstem auditory evoked response or otoacoustic emission should be performed at birth because of increased risk for hearing loss (and per universal newborn hearing screening guidelines). Newborn screening should include measurement of free thyroxine and thyroid-stimulating hormone because many children with trisomy 21 have mildly elevated thyroid-stimulating hormone and normal free thyroxine levels. Because 50% of children with trisomy 21 have congenital heart defects, an echocardiogram should be obtained and read by a pediatric cardiologist even if a normal fetal echocardiogram was obtained. A complete blood cell count is needed to look for hematologic abnormalities, leukemoid reactions, or transient myeloproliferative disorder, which poses an increased risk for leukemia later in life (10%-30%). Leukemia is more common in individuals with trisomy 21 than in the general population, although it is still rare (1%). A bone marrow biopsy would only be indicated if leukemia was suspected. Approximately 3% of children with trisomy 21 have hematologic problems. Magnetic resonance imaging of the brain and renal ultrasonography are not indicated because brain and kidney anomalies are not common in individuals with trisomy 21. The routine serum laboratory values to be followed over time with a diagnosis of trisomy 21 are a complete blood cell count and thyroid function testing, not liver function testing.

Give examples of disease with various modes of inheritance.

autosomal recessive (maple syrup urine disease, sickle cell disease, cystic fibrosis), trinucleotide repeat disorders (fragile X syndrome, myotonic dystrophy), X-linked recessive (hemophilia A and B, Duchenne muscular dystrophy), mitochondrial inheritance (MELAS: mitochondrial encephalomyopathy, lactic acidosis, and stroke-like episodes).

Clinical criteria for neurofibromatosis Type 1.

café-au-lait spots, Lisch nodules, optic gliomas, bony abnormalities, and family history. Bony abnormalities can include: congenital pseudoarthrosis (false joint), most often characterized by unilateral anterolateral angulation (bowing) of the tibia, progressing to thinning of the cortex and pathologic fracture. Long-bone pseudoarthrosis is a characteristic skeletal lesion occurring in 5-10% of infants and toddlers with neurofibromatosis Type 1 (NF1). Up to 80% of children with pseudoarthrosis have NF1; thus, any patient with pseudoarthrosis requires evaluation for NF1. Thinning of the long bone, with or without pseudoarthrosis, is one criterion for NF1. Neurofibromatosis Type 1 (NF1; formerly known as von Recklinghausen disease) is the most common neurocutaneous disease, affecting about 1 in 3,000 individuals worldwide. An estimated 50% of cases are sporadic or de novo autosomal dominant mutations involving the NF1 gene, which encodes the protein neurofibromin. The signs and symptoms of this condition vary widely among affected people, with cutaneous findings (neurofibromatosis Type 1 "spots")—multiple café-au-lait spots, benign cutaneous neurofibromas, and axillary and inguinal freckles—among the most common features. Clinical diagnosis of NF1 requires at least 2 of the 7 criteria indicated below: 1. ≥ 6 café-au-lait spots of ≥ 5 mm in greatest diameter in prepubertal children and ≥ 15 mm in postpubertal children—café-au-lait spots tend to appear in early childhood and increase in size and number as the individual grows older. 2. Freckles in the axillary and inguinal areas—these typically develop later in childhood. 3. ≥ 2 neurofibromas of any type or ≥ 1 plexiform neurofibroma—appearing around puberty, benign peripheral nerve sheath tumors (neurofibromas) that combine Schwann-like cells, fibroblasts, and extracellular matrix may develop on or just under the skin but can also occur in nerves near the spinal cord or along nerves elsewhere in the body. In adolescence or adulthood, some individuals will also develop malignant peripheral nerve sheath tumors, and there is an increased risk of developing other cancers (e.g., leukemia, brain tumor). 4. ≥ 2 Lisch nodules—during childhood, these benign growth iris haramtomas often appear, but they do not interfere with vision. 5. Optic gliomas—these grow along the optic nerve and may have a widely varying impact, from no visual loss to reduced or total vision loss. 6. Sphenoid dysplasia or thinning of the long bone cortex, with or without pseudoarthrosis—this finding may appear in early childhood. 7. 1st degree relative (parent, sibling, or child) with NF1

Newborn ACT sheets and confirmatory algorithms

https://www.ncbi.nlm.nih.gov/books/NBK55827/

Caffey disease

infantile cortical hyperostosis or Caffey disease, a disorder of unknown etiology characterized by extreme irritability, fever, anorexia, and soft tissue swelling associated with subperiosteal cortical thickening of underlying bone.

Patients with homocystinuria have a marfanoid body habitus. What are some characteristics that can distinguish Marfan syndrome from homocystinuria? A Marfan syndrome is due to an inborn error of metabolism, and homocystinuria is not. B Marfan syndrome is autosomal recessive, and homocystinuria is autosomal dominant. C Marfan syndrome has upward displaced lenses, and homocystinuria has downward displaced lenses. D Marfan syndrome is associated with intellectual disabilities, and homocystinuria is not. E Marfan syndrome has a decreased arm span, and homocystinuria does not.

keyboard_arrow_up Answer C Marfan syndrome has upward displaced lenses, and homocystinuria has downward displaced lenses. Explanation Marfan syndrome is an autosomal dominant syndrome. These patients are tall, have normal development, and have an increased arm span. They also have a decreased upper-to-lower body ratio, arachnodactyly, and upward lens subluxation. They are at risk for cardiac disease with aortic dilatation. Patients with homocystinuria, an autosomal recessive disorder affecting the metabolism of methionine, have a similar phenotype to patients with Marfan syndrome. They have a marfanoid body habitus with tall stature, increased arm span, and a decreased upper-to-lower body ratio. Unlike Marfan syndrome, homocystinuria is autosomal recessive and patients with this condition often have intellectual disabilities and a downward lens subluxation. It is due to an inborn error of amino acid metabolism that results in increased homocysteine. Remember the lens direction and disease process according to the following association: Patients with Marfan syndrome have a normal IQ (IQ up) and have upwardly displaced lenses. Patients with homocystinuria have intellectual disabilities (IQ down) and have downwardly displaced lenses.


Conjuntos de estudio relacionados

Bacteriostatic Inhibitors of Protein Synthesis and Aminoglycosides

View Set

`2. Freedom of Information Act (FOIA)

View Set

Andrew Carnegie, Philanthropist with exam review, Communication: Analyzing Bias and Propaganda with exam review, The Crisis, No.1 with exam review, Manufacturing a 14AP4 Picture Tube with exam review, Where Have All the Leaders Gone?, Excerpt with ex...

View Set

Supporting Client Operating Systems Modules 1 - 3 Quiz

View Set